Песок остается на фильтре песок не растворяется в воде: Урок окружающего мира «Вода – растворитель»

Опубликовано в Разное
/
3 Апр 1974

Содержание

Урок окружающего мира «Вода – растворитель»

Цель урока: изучение свойств воды.

Задачи урока: дать представление о воде как растворителе, о растворимых и нерастворимых веществах; познакомить с понятием «фильтр», с простейшими способами определения растворимых и нерастворимых веществ; подготовить доклад на тему «Вода – растворитель».

Оборудование и наглядные пособия: учебники, хрестоматии, тетради для самостоятельной работы; наборы: стаканы пустые и с кипяченой водой; коробочки с поваренной солью, сахаром, речным песком, глиной; чайные ложки, воронки, фильтры из бумажных салфеток; гуашь (акварельные краски), кисти и листы для рефлексии; презентация, выполненная в Power Point, мультимедийный проектор, экран.

ХОД УРОКА

I.  Организационный момент

У. Всем доброе утро! (Слайд 1)
Приглашаю вас на третье заседание школьного научного клуба «Мы и окружающий мир».

II. Сообщение темы и цели урока

Учитель. Сегодня у нас гости, учителя из других школ, которые пришли на заседание клуба. Предлагаю председателю клуба, Порошиной Анастасии, открыть заседание.

Председатель.  Мы сегодня собрались на заседание клуба по теме «Вода – растворитель». Задание всем присутствующим: подготовить доклад на тему «Вода – растворитель». На этом уроке вам вновь предстоит стать исследователями свойств воды. Изучать эти свойства вы будете в своих лабораториях, с помощью «консультантов» – Макаренкова Михаила, Старковой Олеси и Стениной Юлии. Каждая лаборатория должна будет выполнить следующее задание: провести опыты и наблюдения, а в конце заседания обсудить план сообщения «Вода – растворитель».

III. Изучение нового материала

У. С разрешения председателя я хочу сделать первое сообщение. (Слайд 2) Такое же заседание по теме «Вода – растворитель» недавно провели ученики села Мирного. Открыл заседание Костя Погодин, который напомнил всем присутствующим еще об одном удивительном свойстве воды: многие вещества в воде могут распадаться на невидимые мельчайшие частицы, то есть растворяться. Следовательно, для многих веществ вода – хороший растворитель. После этого Маша предложила провести опыты и выявить способы, с помощью которых можно будет получить ответ на вопрос, растворяется вещество в воде или нет.

У. Предлагаю вам на заседании клуба определить растворимость в воде таких веществ, как поваренная соль, сахар, речной песок и глина.
Давайте предположим, какое вещество, по вашему мнению, растворится в воде, а какое не растворится. Выскажите свои предположения, догадки и продолжите высказывание: (Слайд 3)


Предположим …
Допустим …
Возможно …
Что, если …

У. Подумаем вместе, какие гипотезы будем подтверждать. (Слайд 3)
Предположим … (соль растворится в воде)
Допустим … (сахар растворится в воде)
Возможно … (песок не растворится в воде)
Что, если … (глина не растворится в воде)
У. Давайте, и мы проведем опыты, которые помогут нам в этом разобраться.     Перед работой председатель напомнит вам правила при проведении опытов и раздаст карточки, на которых эти правила напечатаны. (Слайд 4)
П. Посмотрите на экран, где записаны правила.

«Правила при проведении опытов»

  1. Необходимо бережно относиться ко всем приборам. Их можно не только разбить, ими можно и пораниться.
  2. Во время работы можно не только сидеть, но и стоять.
  3. Опыт проводит один из учеников (докладчик), остальные молча наблюдают или по просьбе докладчика помогают ему.
  4. Обмен мнениями по результатам проведенного опыта начинается только после того, как докладчик разрешает его начать.
  5. Переговариваться друг с другом нужно тихо, не мешая остальным.
  6. Подходить к столу и проводить замену лабораторного оборудования можно только по разрешению председателя.

IV. Практическая работа

У. Предлагаю председателю выбрать  «консультанта», который прочитает вслух из учебника (с.85) порядок действий при проведении первого опыта. (Слайд 5)

1) П.  Проведите опыт с поваренной солью. Проверьте, растворяется ли в воде поваренная соль.
«Консультант» из каждой лаборатории берет один из подготовленных наборов, и проводит опыт с поваренной солью. В прозрачный стакан наливает кипяченую воду. Всыпает в воду небольшое количество поваренной соли. Группа наблюдает, что происходит с кристалликами соли, и исследует воду на вкус.


Председатель (как в игре КВН) зачитывает один и тот же вопрос каждой группе, а представители от лабораторий отвечают на них.
П. (Слайд 6) Изменилась ли прозрачность воды? (Прозрачность не изменилась)
Изменился ли цвет воды? (Цвет не изменился)
Изменился ли вкус воды? (Вода стала соленой)
Можно ли сказать, что соль исчезла? (Да, она растворилась, исчезла, ее не видно)
У. Сделайте вывод. (Соль растворилась) (Слайд 6)

П. Прошу всех приступить к выполнению второго опыта, для которого необходимо использовать фильтры.

У. Что такое фильтр? (Прибор, устройство или сооружение для очищения жидкостей, газов от твёрдых частиц, примесей.) (Слайд 7)
У. Прочитайте вслух порядок действий при выполнении опыта с фильтром. (Слайд 8)
Учащиеся пропускают воду с солью через фильтр, наблюдают и исследуют воду на вкус.
П. (Слайд 9) Осталась ли соль на фильтре? (На фильтре пищевая соль не осталась)
Изменился ли вкус воды? (Вкус воды не изменился)
Удалось ли очистить воду от соли? (Пищевая соль прошла с водой через фильтр)

У. Сделайте вывод из своих наблюдений. (Соль растворилась в воде) (Слайд 9)
У. Подтвердилась ли ваша гипотеза?
У. Все правильно! Молодцы!
У. Результаты опыта оформите письменно в Тетради для самостоятельной работы (с. 30). (Слайд 10)

2) П. (Слайд 11) Проделаем такой же опыт еще раз, только вместо соли положим чайную ложку сахарного песка.
 «Консультант» из каждой лаборатории берет второй набор и проводит опыт с сахаром. В прозрачный стакан наливает кипяченую воду. Всыпает в воду небольшое количество сахара. Группа наблюдает, что происходит и исследует воду на вкус.


П. (Слайд 12) Изменилась ли прозрачность воды? (Прозрачность воды не изменилась)
Изменился ли цвет воды? (Цвет воды не изменился)
Изменился ли вкус воды? (Вода стала сладкой)
Можно ли сказать, что сахар исчез? (Сахар стал невидимым в воде, вода его растворила)
У. Сделайте вывод. (Сахар растворился) (Слайд 12)

У. Пропустите воду с сахаром через бумажный фильтр. (Слайд 13)
Учащиеся пропускают воду с сахаром через фильтр, наблюдают и исследуют воду на вкус.


П. (Слайд 14) Остался ли сахар на фильтре? (На фильтре сахара не видно)
Изменился ли вкус воды? (Вкус воды не изменился)
Удалось ли очистить воду от сахара? (Воду от сахара очистить не удалось, вместе с водой он прошел через фильтр)
У. Сделайте вывод. (Сахар растворился в воде) (Слайд 14)
У. Подтвердилась ли гипотеза?
У. Верно. Молодцы!
У. Результаты опыта оформите письменно в Тетради для самостоятельной работы. (Слайд 15)

3) П. (Слайд 16) Проверим утверждения и проведем опыт с речным песком

.
У. Прочитайте в учебнике порядок действий при проведении опыта.
Проводят опыт с речным песком. Размешивают в стакане с водой чайную ложку речного песка. Дают смеси отстояться. Наблюдают, что происходит с песчинками и водой.
П. (Слайд 17) Изменилась ли прозрачность воды? (Вода стала мутной, грязной)
Изменился ли цвет воды? (Цвет воды изменился)
Исчезли ли песчинки? (Более тяжелые песчинки опускаются на дно, а мелкие плавают в воде, делая ее мутной)
У. Сделайте вывод. (Песок не растворился) (Слайд 17)

У. (Слайд 18) Пропустите содержимое стакана через бумажный фильтр.
Учащиеся пропускают воду с сахаром через фильтр, наблюдают.
П. (Слайд 19) Что проходит через фильтр, а что остается на нем? (Вода проходит через фильтр, а речной песок остался на фильтре и песчинки хорошо видны)
Очистилась ли вода от песка? (Фильтр помогает очистить воду от частиц, которые в ней не растворяются)
У. Сделайте вывод. (Речной песок в воде не растворился) (Слайд 19)
У. Верным ли было ваше предположение о растворимости песка в воде?
У. Отлично! Молодцы!
У. Результаты опыта оформите письменно в Тетради для самостоятельной работы. (Слайд 20)

4) П. (Слайд 21) Проделайте такой же опыт с кусочком глины.
Проводят опыт с глиной. Размешивают в стакане с водой кусочек глины. Дают смеси отстояться. Наблюдают, что происходит с глиной и водой.
П. (Слайд 22) Изменилась ли прозрачность воды? (Вода стала мутной)
Изменился ли цвет воды? (Да)
Исчезли ли частицы глины? (Более тяжелые частицы опускаются на дно, а мелкие плавают в воде, делая ее мутной)
У. Сделайте вывод. (Глина не растворилась в воде) (Слайд 22)

У. (Слайд 23) Пропустите содержимое стакана через бумажный фильтр.
П. (Слайд 24) Что проходит через фильтр, а что остается на нем? (Вода проходит через фильтр, а не растворившиеся частицы остаются на фильтре.)
Очистилась ли вода от глины? (Фильтр помог очистить воду от частиц, которые не растворились в воде)
У. Сделайте вывод. (Глина не растворяется в воде) (Слайд 24)
У. Гипотеза подтвердилась?
У. Молодцы! Все правильно!

У. Прошу одного из членов группы зачитать выводы, записанные в тетради, всем присутствующим.
У. Есть ли у кого-нибудь дополнения, уточнения?
У. Сделаем выводы из опытов. (Слайд 25)
Все ли вещества растворяются в воде? (Соль, сахарный песок растворились в воде, а песок и глина не растворились.)
Всегда ли с помощью фильтра можно выявить, растворяется вещество в воде или нет? (Растворившиеся в воде вещества проходят через фильтр вместе с водой, а не растворившиеся частицы остаются на фильтре)

У. Прочитайте о растворимости веществ в воде в учебнике (с.87).

У. Сделайте вывод о свойстве воды как растворителя. (Вода – растворитель, но не все вещества в ней растворяются) (Слайд 25)

У. Советую членам клуба прочитать рассказ в хрестоматии «Вода – растворитель» (с.46). (Слайд 26)
Почему же ученым пока не удалось получить абсолютно чистую воду? (Потому что в воде растворены сотни, а может и тысячи разных веществ)

У. Как люди используют свойство воды растворять некоторые вещества?
(Слайд 27) Безвкусная вода становится сладкой или соленой благодаря сахару или соли, так как вода растворяет и приобретает их вкус. Это свойство человек использует, когда готовит пищу: заваривает чай, варит компот, супы, солит и консервирует овощи, заготавливает варенье.
(Слайд 28) Когда мы моем руки, умываемся или купаемся, когда стираем одежду, то используем жидкую воду и ее свойство – растворителя.
(Слайд 29) В воде также растворяются газы, в частности кислород. Благодаря этому в реках, озерах, морях живут рыбы и другие. Соприкасаясь с воздухом, вода растворяет кислород, углекислый газ и другие газы, которые находятся в нем. Для живых организмов, обитающих в воде, например, рыб, очень важен кислород, растворенный в воде. Он им нужен для дыхания. Если бы кислород не растворялся в воде, то водоемы были бы безжизненными. Зная это, люди не забывают насыщать кислородом воду в аквариуме, где живут рыбки, или прорубают зимой проруби в водоемах для улучшения жизни подо льдом.
(Слайд  30) Когда рисуем акварельными красками или гуашью.

У. Обратите внимание на задание, записанное на доске. (Слайд 31) Предлагаю составить коллективный план выступления на тему «Вода – растворитель». Обсудите его в своих лабораториях.
Заслушивание планов по теме «Вода – растворитель», составленных учащимися.
У. Давайте все вместе сформулируем план выступления. (Слайд 31)

Примерный план выступления по теме «Вода – растворитель»

  1. Введение.
  2. Растворение веществ в воде.
  3. Выводы.
  4. Использование людьми свойства воды растворять некоторые вещества.

Экскурсия в «Выставочный зал». (Слайд 32)

У. При подготовке сообщения вы можете использовать дополнительную литературу, подобранную ребятами, помощниками докладчиков по теме нашего заседания. (Обратить внимание учащихся на выставку книг,    интернет – страничек)

V. Итог урока

Какое свойство воды исследовали на заседании клуба? (Свойство воды как растворителя)
К какому выводу мы пришли, исследовав это свойство воды? (Вода – хороший растворитель для некоторых веществ.)
Как вы думаете, трудно быть исследователями?
Что показалось наиболее сложным, интересным?
Пригодятся ли вам знания, приобретенные в ходе исследования этого свойства воды в дальнейшей жизни? (Слайд 33) (Очень важно помнить о том, что вода – растворитель. Вода растворяет соли, среди которых есть как полезные для человека, так и вредные. Поэтому пить воду из источника, если вы не знаете, чист ли он, нельзя. Не зря в народе есть пословица: «Не всякая водица для питья годится».)

VI. Рефлексия

Как мы используем свойство воды растворять некоторые вещества на уроках изобразительного искусства? (Когда рисуем акварельными красками или гуашью)
Предлагаю вам, используя это свойство воды, раскрасить воду в стакане в такой цвет, который наиболее полно соответствует вашему настроению. (Слайд 34)
«Желтый цвет» – радостное, светлое, хорошее настроение.
«Зеленый цвет» – спокойное, уравновешенное.
«Синий цвет» – грустное, печальное, тоскливое настроение.
Покажите свои листы с раскрашенной водой в стакане.

VII. Оценивание

Благодарю председателя, «консультантов» и всех участников заседания за активную работу.

VIII. Домашнее задание

(Слайд 35) Составить по плану доклад для выступления по теме «Вода – растворитель». Лучшие исследовательские работы по этой теме будут отправлены в научный клуб.
(Слайд 36) Спасибо всем за урок. Думаю, что заседание школьного клуба прошло хорошо, и вы можете поделиться своими впечатлениями, написав об этом в клуб любителей природы «Мы и окружающий мир».

Свойства воды — урок. Окружающий мир, 3 класс.

Вода — самое распространённое вещество на нашей планете. Вспомним её свойства.

 

При комнатной температуре вода жидкая. Она принимает форму сосуда, в котором находится.

 

Рис. \(1\). Вода принимает форму сосуда

 

Вода текучая, как и все жидкости. Поэтому на земле есть реки, ручьи и водопады, а в наш дом она может поступать по водопроводу.

 

Рис. \(2\). Вода текучая

 

Вода бесцветная и прозрачная, и мы хорошо видим обитателей водоёма или аквариума.

 

Рис.. \(3\). Вода бесцветная и прозрачная

  

Вода не имеет запаха и вкуса.

 

Вода растворяет многие вещества. Если в воду насыпать соль и перемешать, то соль как бы пропадает. Вода остаётся прозрачной, но становится солёной. Это происходит потому, что частицы соли перемешиваются с частицами воды.

 

Рис. \(4\). Вода — растворитель

 

Растворяются в воде и другие вещества: сахар, уксус, спирт.

 

Но известно много веществ, которые в воде не растворяются. Если смешать с водой песок, то вода станет мутной, а песок через некоторое время осядет на дне сосуда.

 

Рис. \(5\). Вода и песок

 

Не растворяется в воде мел и некоторые жидкости, например, растительное масло и бензин.

 

Для очистки воды от примесей твёрдых веществ используется фильтрование. Мутную воду пропускают через фильтр (специальную бумагу или ткань). На фильтре оседают твёрдые частицы, а вода становится чистой.

 

Рис. \(6\). Фильтрование

 

При нагревании вода расширяется, а при охлаждении сжимается.

 

Рис. \(7\). Сжатие воды при охлаждении

Источники:

Рис. 1. Вода принимает форму сосуда https://image.shutterstock.com/image-photo/water-splash-pitcher-into-glass-600w-158373875.jpg

Рис. 2. Вода текучая https://image.shutterstock.com/image-photo/water-tap-faucet-flow-bathroom-600w-1833087562.jpg

Рис. 3. Вода бесцветная и прозрачная

Рис. 4. Вода — растворитель

Рис. 5. Вода и песок

Рис. 6. Фильтрование © ЯКласс

Рис. 7. Сжатие воды при охлаждении  © ЯКласс

растворитель | Образовательная социальная сеть

Слайд 1

Вода — растворитель

Слайд 2

жидкое вещество, в котором растворяются другие вещества вещество, которое растворилось в растворителе Растворённое вещество Растворитель Великолепный растворитель

Слайд 3

Мы хотим выяснить Многие вещества в воде могут распадаться на невидимые мельчайшие частицы, то есть растворяться. Следовательно, для многих веществ вода – хороший растворитель. Предлагаю провести опыты и выявить способы, с помощью которых можно будет получить ответ на вопрос, растворяется вещество в воде или нет. Что берём? Что наблюдаем? Поваренная соль ? Сах ар ный песок ? Речной песок ? Глина ? От чего зависит растворимость (эксперимент) ?

Слайд 4

Растворимость – это содержание растворенного вещества в насыщенном растворе. Бывают:

Слайд 5

Проведем опыт Наполни прозрачный стакан кипячёной водой. Всыпь в него чайную ложку поваренной соли. Помешивая воду, наблюдай, что происходит с кристалликами соли.

Слайд 6

Соль растворилась в воде. Прозрачность не изменилась. Цвет не изменился. А вот вкус – да! Раствор стал соленый.

Слайд 7

В пустой стакан вставь воронку с фильтром и пропусти через неё воду с солью . Соль вместе с водой прошла через фильтр, она не осталась на фильтре. А вкус после фильтрования прежний. Значит она растворилась.

Слайд 8

Проведем опыт Наполни прозрачный стакан кипячёной водой. Всыпь в него чайную ложку сахарного песка. Помешивая воду, наблюдай, что происходит с кристалликами сахара.

Слайд 9

Сахар растворился в воде. Прозрачность воды не изменилась. Цвет не изменился. Сахара не стало видно в воде. А вот вкус –да!

Слайд 10

В пустой стакан вставь воронку с фильтром и пропусти через неё воду с сахаром. Сахар растворился в воде. Он не остался на фильтре, прошел вместе с водой. А вкус после фильтрования прежний.

Слайд 11

Проведем опыт Размешай в стакане с водой чайную ложку речного песка. Дай смеси отстояться.

Слайд 12

Изменился цвет воды, она стала мутной, грязной. Крупные песчинки легли на дно, мелкие плавают. Песок не растворился.

Слайд 13

В пустой стакан вставь воронку с фильтром и пропусти через неё содержимое. Песок остался на фильтре, вода прошла и очистилась. Фильтр помогает очистить воду от частиц, которые в ней не растворяются.

Слайд 14

Проведем опыт Размешай в стакане с водой чайную ложку глины. Дай смеси отстояться.

Слайд 15

Глина не растворилась в воде, вода мутная, крупные частицы глины упали на дно, а мелкие плавают в воде.

Слайд 16

Пропусти содержимое стакана через бумажный фильтр. Вода проходит через фильтр, а не растворившиеся частицы остаются на фильтре. Фильтр помог очистить воду от частиц, которые не растворились в воде.

Как соль растворяется. Поваренная соль

Мария Роденко
Опыты и эксперименты с водой для дошкольников

Волшебница вода

Вода, пар, лёд – одно и то же вещество!

Налить в блюдце немного воды и поставить на 2-3 часа в морозилку. Что получилось?

Поставить блюдце со льдом на стол. Через сколько времени в нём снова окажется вода? Что произойдёт со льдом – твёрдой водой?

Оставить блюдце с водой на подоконнике на 2-3 дня. Скоро ли она испарится?

Объяснение: вода, пар, лёд – одно и то же вещество, но только в разных состояниях: жидком, твёрдом и газообразном.

2. Можно ли склеить бумагу водой?

Взять два листа бумаги, приложить их друг к другу и попробовать сдвинуть их в разные стороны. Получилось?

Затем намочить листы бумаги водой, приложить их друг к другу, слегка прижать и попробовать сдвинуть их, один в одну сторону, другой в другую.

Объяснение: вода обладает «склеивающим» действием, поэтому бумагу можно склеить водой.

3. Можно ли ускорить испарение воды?

Намочить руки водой из-под крана и быстро-быстро помахать ими. Что чувствуют руки? Почему это происходит?

Объяснение: испарение воды можно ускорить, например, создавая движение воздуха. При этом частички воды испаряются быстрее, унося с собой больше тепла. Поэтому руки ощущают прохладу при взмахах руками.

Капельки — шарики

Взять мелкий песок или муку, сбрызнуть водой. Образуются капельки – шарики.

Обрызгать из пульверизатора листья растения. Какие получились капельки?

Объяснение: частички собирают вокруг себя капельки воды и образуют одну большую капельку-шарик, а когда множество капелек воды попадает на лист растения, они, собираясь вместе, образуют большую каплю-шар или даже маленькую лужицу.

5. Сахар растворяется в воде.

Положить в стакан кусочек сахара и налить чай тоненькой струйкой, стараясь попадать только на сахар. Сахар постепенно будет размываться, а потом… исчезнет? Конечно, нет.

Зачерпнуть ложкой немного чая из стакана и ложку чая из чайника, попробовать и сравнить вкус. Что можно почувствовать, одинаковый ли на вкус чай?

Объяснение: сахар растворяется в воде и смешивается с ней, поэтому вода становится сладкой.

6. Соль растворяется в воде.

В стакан с водой насыпать одну столовую ложку соли и перемешать.

Что произошло? Соль «исчезла»? дать ребёнку попробовать немного воды. Какой стала вода?

Объяснение: соль не исчезла, она растворилась в воде, и вода стала солёной.

Соль выпаривается и кристаллизуется.

В стакан с водой насыпать 2-3 ложки соли. Перемешать до полного растворения. Затем поставить на солнечное место и наблюдать за поведением соли.

Через несколько дней на стенках стакана по мере испарения воды будут появляться кристаллики соли.

Объяснение: вода испаряется, а кристаллики соли оседают на стенках стакана.

8. Песок не растворяется.

Предложить ребёнку сравнить сахар и песок, узнать, что растворяется в воде, а что нет.

Размешать в одном стакане с водой ложку речного песка, а в другом ложку сахара. Дать отстояться.

Посмотреть, что произошло с песчинками и водой.

Объяснение: вода с речным песком стала мутной, грязной. Более тяжёлые песчинки опустились на дно, а мелкие плавают в воде, делая её мутной. Сахар стал невидимым в воде, вода его растворила.

9. Трубочка-пипетка.

Взять два одинаковых стакана, один с водой, а другой – пустой. Опустить трубочку для коктейля в стакан с водой, зажать её сверху указательным пальцем и, не отрывая пальца, перенести её в пустой стакан. Убрать палец от трубочки, и из неё вытечет вода.

Проделав то же самое несколько раз, можно перенести всю воду из одного стакана в другой.

Плавающая рыбка.

На картоне нарисовать и вырезать рыбку. Налить в таз воды. Окунуть зубочистку в жидкое мыло или растительное масло и поставить точку на хвостике рыбки.

Положить рыбку на воду так, чтобы хвостик оказался у края таза, в результате рыбка поплывет.

Для повтора эксперимента нужно помыть и высушить таз.

Объяснение: капелька масла растекается по воде, её частички отталкиваются от частичек воды, и в результате рыбка плывёт.

11. Плавающее яйцо.

Литровую банку наполовину наполнить водой. Ложкой осторожно опустить в неё сырое яйцо и убрать ложку. Как поведёт себя яйцо?

Аккуратно вытащить яйцо. Насыпать в ту же банку с водой полчашки (125 мл) соли и размешать ложкой. Затем опустить яйцо в воду. Как теперь ведёт себя яйцо?

Объяснение: яйцо тонет в чистой воде, потому что оно тяжелее воды. Добавляя соль в воду, мы повышаем её плотность, и поэтому яйцо плавает в ней.

12. Поющие бутылки. Высокие и низкие звуки.

Заполнить водой 3 одинаковые бутылки до половины, а затем перелить половину воды из одной в другую. Поднести бутылку к губам, подуть поверх горлышка, чтобы услышать поющий звук. Подуть над другими бутылками, одинаково ли они поют?

Расположить бутылки в порядке высоты звука.

Объяснение: выдыхаемый воздух над бутылкой заставляет вибрировать воздух внутри неё. Высота звука зависит от количества воздуха внутри бутылки.

13. Радужная бумага.

Глубокую миску наполовину наполнить водой. Аккуратно капнуть каплю прозрачного лака для ногтей, лак растечётся по поверхности воды.

Погрузить в воду кусочек чёрной двухсторонней бумаги, вынуть его и просушить. На сухой бумаге можно увидеть радужные разводы.

Объяснение: лак образует на поверхности воды тонкую плёнку. Плёнка обволакивает бумагу, и свет, попадая на неё, отражается, образуя радужный узор.

14. Распускающиеся цветы.

Нарисовать и вырезать из цветной бумаги цветы с длинными лепестками. При помощи карандаша закрутить лепестки к центру цветка.

Налить в таз воду и опустить на неё цветочки. Они начнут расправлять лепесточки и распускаться.

Объяснение: при контакте с водой бумага намокает, становится тяжелее, и лепестки цветка начинают распускаться.

15. Не выливающаяся вода.

Налить до краёв в стакан воды. Положить сверху открытку или квадрат из плотного картона. Прижимая открытку к стакану рукой, перевернуть его вверх дном над раковиной.

Убрать руку. Что произошло?

Объяснение: открытка не падает, и вода не выливается, потому что воздух, находящийся в стакане, давит на неё снизу и прижимает её к краю стакана. Вода выльется, если открытку сдвинуть.

16. Невидимые чернила.

Выдавить сок из половинки лимона в чашку, добавить такое же количество воды. Обмакнуть ватную палочку в приготовленный раствор. Написать ей что-нибудь на листе бумаги.

Когда надпись высохнет, нагреть бумагу над включённой настольной лампой. На бумаге проявятся невидимые ранее слова.

17.
Прыгающие зёрнышки.

В стакан налить газированную воду и опустить 6 зёрнышек риса.

Подождать несколько секунд и понаблюдать сквозь стакан, как зёрнышки начнут прыгать.

Объяснение: рис чуть тяжелее воды, когда он попадает в стакан, начинает опускаться. Пузырьки газа садятся на него и поднимают вверх. На поверхности пузырьки лопаются, и газ улетучивается. Отяжелевший рис вновь опускается вниз. Он будет опускаться, и подниматься, пока вода не «выдохнется».

ПОВАРЕННАЯ СОЛЬ –хлорид натрия NaCl. Умеренно растворяется в воде, растворимость мало зависит от температуры: коэффициент растворимости NaCl (в г на 100 г воды) равен 35,9 при 20° С и 38,1 при 80° С. Растворимость хлорида натрия существенно снижается в присутствии хлороводорода, гидроксида натрия, солей – хлоридов металлов. Растворяется в жидком аммиаке, вступает в реакции обмена. Плотность NaCl – 2,165 г/см 3 , температура плавления 800,8° С, температура кипения 1465° С.

Раньше говаривали: «Соль всему голова, без соли и жито – трава»; «Один глаз на полицу (где хлеб), другой – в солоницу (солонку)», и еще: «Без хлеба не сытно, без соли не сладко»… Бурятская народная мудрость гласит: «Собираясь пить чай, клади в него щепотку соли; от нее быстрее усваивается пища, исчезнут болезни желудка».

Вряд ли мы узнаем, когда впервые наши далекие предки вкусили соль: нас отделяют от них десять-пятнадцать тысяч лет. Тогда еще не было посуды для приготовления пищи, все растительные продукты люди вымачивали в воде и подпекали на тлеющих углях, а мясо, насаженное на палки, обжаривали в пламени костра. «Поваренной солью» первобытных людей наверняка была зола, которая неизбежно попадала в пищу во время ее приготовления. Зола содержит поташ – карбонат калия K 2 CO 3 , который в местах, удаленных от морей и соленых озер, долгое время служил пищевой приправой.

Возможно, однажды, за неимением пресной воды, мясо либо корни и листья растений были замочены в соленой морской или озерной воде, и еда оказалась вкуснее обычной. Может быть, добытое впрок мясо, чтобы защитить его от хищных птиц и насекомых, люди спрятали в морскую воду, а потом обнаружили, что оно приобрело приятный вкус. Наблюдательные охотники первобытных племен могли заметить, что животные любят лизать солонцы – белые кристаллы каменной соли, выступающие кое-где из-под земли, и попробовали добавлять соль в пищу. Могли быть и другие случаи первого знакомства людей с этим удивительным веществом.

Чистая поваренная соль, или хлорид натрия NaCl – бесцветное негигроскопичное (не поглощающее влагу из воздуха) кристаллическое вещество, растворимое в воде и плавящееся при 801° С. В природе хлорид натрия встречается в виде минерала галита – каменной соли. Слово «галит» происходит от греческого «галос», означающего и «соль», и «море». Основная масса галита чаще всего находится на глубине 5 км под поверхностью земли. Однако давление слоя горных пород, расположенных над пластом соли, превращает ее в вязкую, пластичную массу. «Всплывая» в местах пониженного давления кроющих пород, пласт соли образует соляные «купола», выходящие в ряде мест наружу.

Природный галит редко бывает чисто белого цвета. Чаще он буроватый или желтоватый из-за примесей соединений железа. Встречаются, но очень редко, кристаллы галита голубого цвета. Это означает, что они долгое время в глубинах земли находились по соседству с породами, содержащими уран, и подверглись радиоактивному облучению.

В лаборатории тоже можно получить синие кристаллы хлорида натрия. Для этого не потребуется облучения; просто в плотно закрытом сосуде надо нагреть смесь поваренной соли NaCl и небольшого количества металлического натрия Na. Металл способен растворяться в соли. Когда атомы натрия проникают в кристалл, состоящий из катионов Na + и анионов Cl – , они «достраивают» кристаллическую решетку, занимая подходящие места и превращаясь в катионы Na + . Освободившиеся электроны располагаются в тех местах кристалла, где полагалось бы находиться хлорид-анионам Cl –? . Такие необычные места внутри кристалла, занятые электронами вместо ионов, называют «вакансиями».

При охлаждении кристалла некоторые вакансии объединяются, это и служит причиной появления синей окраски. Кстати, при растворении в воде синего кристалла соли образуется бесцветный раствор – совсем как из обычной соли.

Греческий поэт Гомер (VIII в. до н.э.), написавший Илиаду и Одиссею , называл поваренную соль «божественной». В те времена она ценилась выше золота: ведь, как гласила пословица, «без золота прожить можно, а без соли — нельзя». Из-за месторождений каменной соли происходили военные столкновения, а иногда нехватка соли вызывала «соляные бунты».

На столах императоров, царей, королей и шахов стояли солонки из золота, и заведовал ими особо доверенный человек — солоничий. Воинам часто платили жалованье солью, а чиновники получали солевой паек. Как правило, соляные источники были собственностью властителей и коронованных особ. В Библии есть выражение «пьет соль от дворца царского», означающее человека, получающего содержание от царя.

Соль издавна была символом чистоты и дружбы. «Вы – соль земли» – говорил Христос своим ученикам, имея в виду их высокие нравственные качества. Соль употреблялась при жертвоприношениях, новорожденных детей у древних евреев посыпали солью, а в католических церквах при крещении в ротик младенца клали кристаллик соли.

В обычае арабов было при утверждении торжественных договоров подавать сосуд с солью, из которого в знак доказательства и гарантии постоянной дружбы лица, заключившие договор – «завет соли» – съедали по несколько ее крупинок. «Съесть вместе пуд соли» – у славян значит хорошо узнать друг друга и подружиться. По русскому обычаю, когда подносят гостям хлеб-соль, то тем самым желают им здоровья.

Поваренная соль – не только пищевой продукт, но издавна распространенный консервант, ее применяли при обработке кожевенного и мехового сырья. А в технике она до сих пор является исходным сырьем для получения практически всех соединений натрия, в том числе соды.

Поваренная соль входила и в состав самых древних лекарств, ей приписывали целебные свойства, очищающее и дезинфицирующее действие, причем с давних пор подмечено, что поваренная соль разных месторождений имеет разные биологические свойства: самая в этом отношении полезная – морская. В Лечебнике-травнике , изданном в России в 17 в., написано: «Две сути соли, одну копали из горы, а другую находили в море, а которая из моря, та лутчи, а кроме морской соли та лутчи, которая бела».

Однако в употреблении соли надо соблюдать меру. Известно, что средний европеец ежедневно поглощает с пищей до 15 г соли, в то время как средний японец – около 40 г. Как раз японцы и держат мировое первенство по числу больных гипертонией – болезнью, одна из причин которой состоит в том, что в организме задерживается больше жидкости, чем ему необходимо. Клетки разбухают от ее излишка, сжимают кровеносные сосуды, поэтому повышается кровяное давление, от чего и сердце начинает работать с перегрузкой. Трудно становится и почкам, очищающим организм от избытка катионов натрия.

Ни одно растение не может расти на почве, покрытой солью, солончаки всегда были символом земли бесплодной и необитаемой. Когда властитель Священной Римской империи Фридрих I Барбаросса разрушил в 1155 Милан в Италии, то велел посыпать руины поверженного города солью в знак его полного уничтожения… У разных народов во все времена рассыпать соль значило накликать беду и потерять здоровье.

В древности люди использовали несколько способов добычи поваренной соли: естественное испарение морской воды в «соляных садках», где выпадал хлорид натрия NaCl – «морская» соль, вываривание воды соленых озер с получением «выварочной» соли, и выламывание «каменной» соли в подземных рудниках. Все эти способы дают соль с примесями хлорида магния MgCl 2 ·6 H 2 O, сульфатов калия K 2 SO 4 и магния MgSO 4 ·7H 2 O и бромида магния MgBr 2 ·6H 2 O, содержание которых достигает 8–10 %.

В морской воде в среднем на 1 л приходится до 30 г различных солей, на долю поваренной соли приходится 24 г. Технология получения хлорида натрия NaCl из морской и озерной воды всегда была довольно примитивной.

Например, в конце «бронзового века» – за три, три с половиной тысячи лет до нашей эры – древние солевары обливали бревна морской водой, а потом сжигали их и из золы выбирали соль. Позднее соленые воды стали выпаривать на больших противнях, а для удаления примесей добавляли кровь животных, собирая образующуюся пену. Примерно с конца 16 в. растворы соли очищали и концентрировали, пропуская через башни, заполненные соломой и ветками кустарников. Выпаривание раствора соли на воздухе производили и совсем примитивным способом, сливая рассол по стене, сложенной из связок хвороста и соломы.

Солеварение, старейшее из химических ремесел, возникло на Руси, по-видимому, в начале 7 века. Соляные промыслы принадлежали монахам, которым благоволили русские цари, с них даже не взимался налог на продаваемую соль. Выварка соли приносила монастырям огромные прибыли. Рассолы добывали не только из озер, но и из подземных соляных источников; буровые скважины, которые для этого строили, в 15 в. достигали длины 60–70 м. В скважины опускали трубы, изготовленные из цельного дерева, а упаривали рассолы в железных противнях на дровяной топке. В 1780 в России таким способом было выварено более ста тысяч тонн соли…

В настоящее время поваренную соль добывают из отложений соляных озер и на месторожденияхкаменной соли – галита.

Поваренная соль – не только важная пищевая приправа, но и химическое сырье: из нее получают гидроксид натрия, соду, хлор.

Людмила Аликберова

Соль можно определить как соединение, которое образуется в результате реакции между кислотой и основанием, но не является водой. В данном разделе будут рассмотрены те свойства солей, которые связаны с ионными равновесиями.

реакции солей в воде

Несколько позже будет показано, что растворимость-это относительное понятие. Однако для целей предстоящего обсуждения мы можем грубо подразделить все соли на растворимые и нерастворимые в воде.

Некоторые соли при растворении в воде образуют нейтральные растворы. Другие соли образуют кислые либо щелочные растворы. Это обусловлено протеканием обратимой реакции между ионами соли и водой, в результате которой образуются сопряженные кислоты либо основания. Окажется ли раствор соли нейтральным, кислым или щелочным-зависит от типа соли. В этом смысле существуют четыре типа солей.

Соли, образуемые сильными кислотами и слабыми основаниями. Соли этого типа при растворении в воде образуют кислый раствор. В качестве примера приведем хлорид аммония Nh5Cl. При растворении этой соли в воде ион аммония действует как

Избыточное количество ионов h4O+, образуемое в этом процессе, обусловливает кислые свойства раствора.

Соли, образуемые слабой кислотой и сильным основанием. Соли этого типа при растворении в воде образуют щелочный раствор. В качестве примера приведем ацетат натрия Ch4COONa1 Ацетат-ион действует как основание, акцептируя протон у воды, которая выступает в этом случае в роли кислоты:

Избыточное количество ионов ОН-, образующихся в этом процессе, обусловливает щелочные свойства раствора.

Соли, образуемые сильными кислотами и сильными основаниями. При растворении в воде солей этого типа образуется нейтральный раствор. В качестве примера приведем хлорид натрия NaCl. При растворении в воде эта соль полностью ионизируется, и, следовательно, концентрация ионов Na+ оказывается равной концентрации ионов Cl-. Поскольку ни тот, ни другой ион не вступает в кислотно-основные реакции с водой, в растворе не происходит образования избыточного количества ионов h4O+ либо ОН. Поэтому раствор оказывается нейтральным.

Соли, образуемые слабыми кислотами и слабыми основаниями. Примером солей такого типа является ацетат аммония. При растворении в воде ион аммония реагирует с водой как кислота, а ацетат-ион реагирует с водой как основание. Обе эти реакции описаны выше. Водный раствор соли, образованной слабой кислотой и слабым основанием, может быть слабокислым, слабощелочным либо нейтральным в зависимости от относительных концентраций ионов h4O+ и ОН-, образуемых в результате реакций катионов и анионов соли с водой. Это зависит от соотношения между значениями констант диссоциации катиона и аниона.





































Назад Вперёд

Внимание! Предварительный просмотр слайдов используется исключительно в ознакомительных целях и может не давать представления о всех возможностях презентации. Если вас заинтересовала данная работа, пожалуйста, загрузите полную версию.

Цель урока: изучение свойств воды.

Задачи урока: дать представление о воде как растворителе, о растворимых и нерастворимых веществах; познакомить с понятием «фильтр», с простейшими способами определения растворимых и нерастворимых веществ; подготовить доклад на тему «Вода – растворитель».

Оборудование и наглядные пособия: учебники, хрестоматии, тетради для самостоятельной работы; наборы: стаканы пустые и с кипяченой водой; коробочки с поваренной солью, сахаром, речным песком, глиной; чайные ложки, воронки, фильтры из бумажных салфеток; гуашь (акварельные краски), кисти и листы для рефлексии; презентация, выполненная в Power Point, мультимедийный проектор, экран.

ХОД УРОКА

I. Организационный момент

У. Всем доброе утро! (Слайд 1)
Приглашаю вас на третье заседание школьного научного клуба «Мы и окружающий мир».

II. Сообщение темы и цели урока

Учитель. Сегодня у нас гости, учителя из других школ, которые пришли на заседание клуба. Предлагаю председателю клуба, Порошиной Анастасии, открыть заседание.

Председатель. Мы сегодня собрались на заседание клуба по теме «Вода – растворитель». Задание всем присутствующим: подготовить доклад на тему «Вода – растворитель». На этом уроке вам вновь предстоит стать исследователями свойств воды. Изучать эти свойства вы будете в своих лабораториях, с помощью «консультантов» – Макаренкова Михаила, Старковой Олеси и Стениной Юлии. Каждая лаборатория должна будет выполнить следующее задание: провести опыты и наблюдения, а в конце заседания обсудить план сообщения «Вода – растворитель».

III. Изучение нового материала

У. С разрешения председателя я хочу сделать первое сообщение. (Слайд 2) Такое же заседание по теме «Вода – растворитель» недавно провели ученики села Мирного. Открыл заседание Костя Погодин, который напомнил всем присутствующим еще об одном удивительном свойстве воды: многие вещества в воде могут распадаться на невидимые мельчайшие частицы, то есть растворяться. Следовательно, для многих веществ вода – хороший растворитель. После этого Маша предложила провести опыты и выявить способы, с помощью которых можно будет получить ответ на вопрос, растворяется вещество в воде или нет.
У. Предлагаю вам на заседании клуба определить растворимость в воде таких веществ, как поваренная соль, сахар, речной песок и глина.
Давайте предположим, какое вещество, по вашему мнению, растворится в воде, а какое не растворится. Выскажите свои предположения, догадки и продолжите высказывание: (Слайд 3)

У. Подумаем вместе, какие гипотезы будем подтверждать. (Слайд 3)
Предположим … (соль растворится в воде)
Допустим … (сахар растворится в воде)
Возможно … (песок не растворится в воде)
Что, если … (глина не растворится в воде)
У. Давайте, и мы проведем опыты, которые помогут нам в этом разобраться. Перед работой председатель напомнит вам правила при проведении опытов и раздаст карточки, на которых эти правила напечатаны. (Слайд 4)
П. Посмотрите на экран, где записаны правила.

«Правила при проведении опытов»

  1. Необходимо бережно относиться ко всем приборам. Их можно не только разбить, ими можно и пораниться.
  2. Во время работы можно не только сидеть, но и стоять.
  3. Опыт проводит один из учеников (докладчик), остальные молча наблюдают или по просьбе докладчика помогают ему.
  4. Обмен мнениями по результатам проведенного опыта начинается только после того, как докладчик разрешает его начать.
  5. Переговариваться друг с другом нужно тихо, не мешая остальным.
  6. Подходить к столу и проводить замену лабораторного оборудования можно только по разрешению председателя.

IV. Практическая работа

У. Предлагаю председателю выбрать «консультанта», который прочитает вслух из учебника (с.85) порядок действий при проведении первого опыта. (Слайд 5)

1) П. Проведите опыт с поваренной солью . Проверьте, растворяется ли в воде поваренная соль.
«Консультант» из каждой лаборатории берет один из подготовленных наборов, и проводит опыт с поваренной солью. В прозрачный стакан наливает кипяченую воду. Всыпает в воду небольшое количество поваренной соли. Группа наблюдает, что происходит с кристалликами соли, и исследует воду на вкус.
Председатель (как в игре КВН) зачитывает один и тот же вопрос каждой группе, а представители от лабораторий отвечают на них.
П. (Слайд 6) Изменилась ли прозрачность воды? (Прозрачность не изменилась)
Изменился ли цвет воды? (Цвет не изменился)
Изменился ли вкус воды? (Вода стала соленой)
Можно ли сказать, что соль исчезла? (Да, она растворилась, исчезла, ее не видно)
У. Сделайте вывод. (Соль растворилась) (Слайд 6)

П. Прошу всех приступить к выполнению второго опыта, для которого необходимо использовать фильтры.
У. Что такое фильтр? (Прибор, устройство или сооружение для очищения жидкостей, газов от твёрдых частиц, примесей.) (Слайд 7)
У. Прочитайте вслух порядок действий при выполнении опыта с фильтром. (Слайд 8)
Учащиеся пропускают воду с солью через фильтр, наблюдают и исследуют воду на вкус.
П. (Слайд 9) Осталась ли соль на фильтре? (На фильтре пищевая соль не осталась)
Изменился ли вкус воды? (Вкус воды не изменился)
Удалось ли очистить воду от соли? (Пищевая соль прошла с водой через фильтр)
У. Сделайте вывод из своих наблюдений. (Соль растворилась в воде) (Слайд 9)
У. Подтвердилась ли ваша гипотеза?
У. Все правильно! Молодцы!
У. Результаты опыта оформите письменно в Тетради для самостоятельной работы (с. 30). (Слайд 10)

2) П. (Слайд 11) Проделаем такой же опыт еще раз, только вместо соли положим чайную ложку сахарного песка .
«Консультант» из каждой лаборатории берет второй набор и проводит опыт с сахаром. В прозрачный стакан наливает кипяченую воду. Всыпает в воду небольшое количество сахара. Группа наблюдает, что происходит и исследует воду на вкус.
П. (Слайд 12) Изменилась ли прозрачность воды? (Прозрачность воды не изменилась)
Изменился ли цвет воды? (Цвет воды не изменился)
Изменился ли вкус воды? (Вода стала сладкой)
Можно ли сказать, что сахар исчез? (Сахар стал невидимым в воде, вода его растворила)
У. Сделайте вывод. (Сахар растворился) (Слайд 12)

У. Пропустите воду с сахаром через бумажный фильтр. (Слайд 13)
Учащиеся пропускают воду с сахаром через фильтр, наблюдают и исследуют воду на вкус.
П. (Слайд 14) Остался ли сахар на фильтре? (На фильтре сахара не видно)
Изменился ли вкус воды? (Вкус воды не изменился)
Удалось ли очистить воду от сахара? (Воду от сахара очистить не удалось, вместе с водой он прошел через фильтр)
У. Сделайте вывод. (Сахар растворился в воде) (Слайд 14)
У. Подтвердилась ли гипотеза?
У. Верно. Молодцы!
У. Результаты опыта оформите письменно в Тетради для самостоятельной работы. (Слайд 15)

3) П. (Слайд 16)Проверим утверждения и проведем опыт с речным песком .
У. Прочитайте в учебнике порядок действий при проведении опыта.
Проводят опыт с речным песком. Размешивают в стакане с водой чайную ложку речного песка. Дают смеси отстояться. Наблюдают, что происходит с песчинками и водой.
П. (Слайд 17) Изменилась ли прозрачность воды? (Вода стала мутной, грязной)
Изменился ли цвет воды? (Цвет воды изменился)
Исчезли ли песчинки? (Более тяжелые песчинки опускаются на дно, а мелкие плавают в воде, делая ее мутной)
У. Сделайте вывод. (Песок не растворился) (Слайд 17)

У. (Слайд 18) Пропустите содержимое стакана через бумажный фильтр.
Учащиеся пропускают воду с сахаром через фильтр, наблюдают.
П. (Слайд 19) Что проходит через фильтр, а что остается на нем? (Вода проходит через фильтр, а речной песок остался на фильтре и песчинки хорошо видны)
Очистилась ли вода от песка? (Фильтр помогает очистить воду от частиц, которые в ней не растворяются)
У. Сделайте вывод. (Речной песок в воде не растворился) (Слайд 19)
У. Верным ли было ваше предположение о растворимости песка в воде?
У. Отлично! Молодцы!
У. Результаты опыта оформите письменно в Тетради для самостоятельной работы. (Слайд 20)

4) П. (Слайд 21) Проделайте такой же опыт с кусочком глины.
Проводят опыт с глиной. Размешивают в стакане с водой кусочек глины. Дают смеси отстояться. Наблюдают, что происходит с глиной и водой.
П. (Слайд 22) Изменилась ли прозрачность воды? (Вода стала мутной)
Изменился ли цвет воды? (Да)
Исчезли ли частицы глины? (Более тяжелые частицы опускаются на дно, а мелкие плавают в воде, делая ее мутной)
У. Сделайте вывод. (Глина не растворилась в воде) (Слайд 22)

У. (Слайд 23) Пропустите содержимое стакана через бумажный фильтр.
П. (Слайд 24)Что проходит через фильтр, а что остается на нем? (Вода проходит через фильтр, а не растворившиеся частицы остаются на фильтре.)
Очистилась ли вода от глины? (Фильтр помог очистить воду от частиц, которые не растворились в воде)
У. Сделайте вывод. (Глина не растворяется в воде) (Слайд 24)
У. Гипотеза подтвердилась?
У. Молодцы! Все правильно!

У. Прошу одного из членов группы зачитать выводы, записанные в тетради, всем присутствующим.
У. Есть ли у кого-нибудь дополнения, уточнения?
У. Сделаем выводы из опытов. (Слайд 25)
Все ли вещества растворяются в воде? (Соль, сахарный песок растворились в воде, а песок и глина не растворились.)
Всегда ли с помощью фильтра можно выявить, растворяется вещество в воде или нет? (Растворившиеся в воде вещества проходят через фильтр вместе с водой, а не растворившиеся частицы остаются на фильтре)

У. Прочитайте о растворимости веществ в воде в учебнике (с.87).

У. Сделайте вывод о свойстве воды как растворителя. (Вода – растворитель, но не все вещества в ней растворяются) (Слайд 25)

У. Советую членам клуба прочитать рассказ в хрестоматии «Вода – растворитель» (с.46). (Слайд 26)
Почему же ученым пока не удалось получить абсолютно чистую воду? (Потому что в воде растворены сотни, а может и тысячи разных веществ)

У. Как люди используют свойство воды растворять некоторые вещества?
(Слайд 27) Безвкусная вода становится сладкой или соленой благодаря сахару или соли, так как вода растворяет и приобретает их вкус. Это свойство человек использует, когда готовит пищу: заваривает чай, варит компот, супы, солит и консервирует овощи, заготавливает варенье.
(Слайд 28) Когда мы моем руки, умываемся или купаемся, когда стираем одежду, то используем жидкую воду и ее свойство – растворителя.
(Слайд 29) В воде также растворяются газы, в частности кислород. Благодаря этому в реках, озерах, морях живут рыбы и другие. Соприкасаясь с воздухом, вода растворяет кислород, углекислый газ и другие газы, которые находятся в нем. Для живых организмов, обитающих в воде, например, рыб, очень важен кислород, растворенный в воде. Он им нужен для дыхания. Если бы кислород не растворялся в воде, то водоемы были бы безжизненными. Зная это, люди не забывают насыщать кислородом воду в аквариуме, где живут рыбки, или прорубают зимой проруби в водоемах для улучшения жизни подо льдом.
(Слайд 30) Когда рисуем акварельными красками или гуашью.

У. Обратите внимание на задание, записанное на доске. (Слайд 31) Предлагаю составить коллективный план выступления на тему «Вода – растворитель». Обсудите его в своих лабораториях.
Заслушивание планов по теме «Вода – растворитель», составленных учащимися.
У. Давайте все вместе сформулируем план выступления. (Слайд 31)

Примерный план выступления по теме «Вода – растворитель»

  1. Введение.
  2. Растворение веществ в воде.
  3. Выводы.
  4. Использование людьми свойства воды растворять некоторые вещества.

Экскурсия в «Выставочный зал». (Слайд 32)

У. При подготовке сообщения вы можете использовать дополнительную литературу, подобранную ребятами, помощниками докладчиков по теме нашего заседания. (Обратить внимание учащихся на выставку книг, интернет – страничек)

V. Итог урока

Какое свойство воды исследовали на заседании клуба? (Свойство воды как растворителя)
К какому выводу мы пришли, исследовав это свойство воды? (Вода – хороший растворитель для некоторых веществ.)
Как вы думаете, трудно быть исследователями?
Что показалось наиболее сложным, интересным?
Пригодятся ли вам знания, приобретенные в ходе исследования этого свойства воды в дальнейшей жизни? (Слайд 33) (Очень важно помнить о том, что вода – растворитель. Вода растворяет соли, среди которых есть как полезные для человека, так и вредные. Поэтому пить воду из источника, если вы не знаете, чист ли он, нельзя. Не зря в народе есть пословица: «Не всякая водица для питья годится».)

VI. Рефлексия

Как мы используем свойство воды растворять некоторые вещества на уроках изобразительного искусства? (Когда рисуем акварельными красками или гуашью)
Предлагаю вам, используя это свойство воды, раскрасить воду в стакане в такой цвет, который наиболее полно соответствует вашему настроению. (Слайд 34)
«Желтый цвет» – радостное, светлое, хорошее настроение.
«Зеленый цвет» – спокойное, уравновешенное.
«Синий цвет» – грустное, печальное, тоскливое настроение.
Покажите свои листы с раскрашенной водой в стакане.

VII. Оценивание

Благодарю председателя, «консультантов» и всех участников заседания за активную работу.

VIII. Домашнее задание

Солевой раствор является очень эффективным антисептиком, и поэтому получил широкое распространение в качестве средства первой помощи и домашнего терапевтического лечения при различных инфекциях.

Приготовление солевого раствора

Это очень простой процесс, так как соль очень хорошо растворима в воде. Следует принять во внимание, что в лечебных целях солевые растворы можно готовить только из чистой морской или поваренной соли.

  1. Для приготовления раствора необходимо взять емкость с нужным количеством воды.
  2. Добавить соль в заданных пропорциях.
  3. После этого тщательно перемешать раствор до полного растворения соли. В горячей воде соль растворяется значительно быстрее.

Солевой раствор для полоскания

При воспалении верхних дыхательных путей для промывания носоглотки и полоскания горла используют солевой раствор в пропорциях 1/2 чайной ложки соли на стакан воды. При лечении ангины для большей эффективности можно добавить в раствор 1/2 чайной ложки соды. Этот раствор используют для полоскания полости рта при заболеваниях десен или при других воспалительных процессах в полости рта.

Солевой раствор для примочек и компрессов

Для наружного лечения иногда используют солевые компрессы или делают примочки, способствующие заживлению небольших ран и рассасыванию гематом. Для этого используют раствор концентрацией 3-5 ложек соли на стакан воды.

При серьезных ушибах можно делать теплые компрессы с раствором соли в воде в пропорции 100 граммов на 1/2 литра.

Насыщенный раствор соли

Для приготовления насыщенного раствора, соль постепенно добавляют в воду и помешивают до полного растворения.

Так продолжается до прекращения растворения соли. Если соль при продолжительном размешивании не растворяется в воде, а оседает на дне стакана, значит, ее содержание в воде достигло максимума возможного в нормальных условиях. Такой раствор называется насыщенным. Его можно использовать, например, для выращивания солевых кристаллов.

Какие вещества растворяются в воде соль мука песок масло

Главная » Разное » Какие вещества растворяются в воде соль мука песок масло

План-конспект занятия по окружающему миру (старшая группа) на тему: Вода – растворитель. Очищение воды.

Вода – растворитель. Очищение воды.

Цель: закрепить понимание того, что вещества в воде не исчезают, а растворяются.

Задачи:

  1. Выявить вещества, которые растворяются в воде.
  2. Выявить вещества, которые не растворяются в воде.
  3. Познакомить со способом очистки воды – фильтрованием.
  4. Создать условия для выявления и проверки различных способов очистки воды.
  5. Закрепить знания о правилах безопасного поведения при работе с различными веществами.
  6. Развивать умение ставить перед собой цель, планировать свою работу.
  7. Развивать логическое мышление путем моделирования проблемных ситуаций и их решения.
  8. Закреплять в речи детей слова : фильтр, водопровод, воронка.
  9. Воспитывать аккуратность и безопасное поведение при работе с различными веществами.

Материалы:

Сосуды разного размера, вода, стиральный порошок, соль, мука, сахар, шампунь, песок, растительное масло, марганцовка, чай, ложки, бумага, марля, фильтры бумажные, передники.

Гипотеза:

  1. Если вещество поместить в воду, то оно растворится и изменит свойства воды.
  2. Если вещество не растворится, то и свойства воды останутся прежними
  3. Загрязненную воду можно отфильтровать.

Описание эксперимента.

В гости к детям пришла капелька и принесла много различных веществ. Она просит помочь ее разобраться в том, что произойдет с водой при взаимодействии с ними. Перед тем, как определять, что это за вещества, дети вспоминают правила работы с ними: нельзя пробовать вещества на вкус – есть опасность отравиться, нюхать нужно осторожно, направляя запах от стакана ладошкой, так как вещества могут быть очень едкими и можно обжечь дыхательные пути.

Сначала дети выясняют, что изменится, если растворить исследуемые вещества в воде. Дети растворяют различные вещества в разных сосудах.

Чистая вода + вещество

Что получилось

Соль

Быстро растворилась в воде, раствор остался прозрачным.

Сахар

Быстро растворился в воде, раствор остался прозрачным.

Стиральный порошок

Легко растворился, но раствор стал мутноватым и начал пениться при интенсивном помешивании.

Песок

Придал воде коричневатый цвет (вода стала грязной) и осел в виде осадка на дне.

Мука

Растворилась в воде, но вода стала мутной, часть муки осела в виде осадка

Шампунь

Легко растворился, но раствор стал мутноватым и начал пениться при интенсивном помешивании.

Растительное масло

Масло не растворилось в воде. Оно плавает по ее поверхности в виде пленки. Если раствор взболтать, то масло приобретает на некоторое время вид мелких капелек, которые затем снова преобразуются в пленку.

Марганцовка

Быстро растворилась и при этом изменила цвет раствора на красный.

Чай

Быстро изменил цвет раствора на коричневый, но при этом сам не растворился.

Камень

Осел на дно, цвет воды не изменил, но заметно ее вытеснил (воды словно стало больше )

Можно ли теперь воду очистить от разных веществ? Как это можно сделать? Полина говорит, что воду можно отфильтровать.

Дети фильтруют воду. Результаты заносятся в таблицу.

Раствор

Что получилось после фильтрования

Чистая вода + соль

Практически не от фильтровался.

Чистая вода +сахар

Практически не от фильтровался.

Чистая вода +стиральный порошок

Вода все равно мутноватая, на фильтре видны следы крупинок порошка.

Чистая вода +песок

Вода в стакане кажется почти чистой, на фильтре много песчинок.

Чистая вода +мука

Раствор по-прежнему имеет мутноватый цвет. На фильтре видны следы клейковины.

Чистая вода +шампунь

Практически не от фильтровался.

Чистая вода +растительное масло

От фильтровалось быстро, на фильтре видны его следы

Чистая вода +марганцовка

Вода после фильтрования заметно посветлела, но чистой на стала.

Чистая вода +чай

Практически не от фильтровался. На фильтре оставил темные следы и чаинки. Получившийся отфильтрованный раствор коричневого цвета.

Чистая вода + камень

Вода в стакане чистая, камень остался лежать в воронке.

Вывод:

  1. Вода растворяет многие вещества, и не все из них удается отфильтровать в бытовых условиях.
  2. Даже после фильтрования загрязненную воду нельзя употреблять в пищу не кипяченой.
  3. Загрязненной водой после ее фильтрования можно мыть руки, стирать белье и использовать для различных хозяйственных нужд.

Почему соль растворяется в воде, а не в масле?

MarianVejcik / iStock / GettyImages

Like Dissolves Like

Основной принцип, определяющий, будет ли вещество растворяться в другом, — «подобное растворяется в подобном». Это означает, что если молекулы двух веществ обладают общим химическим свойством, одно может растворяться в другом. Иначе не будет. Это является основой для ответа на вопрос: «почему соль растворяется в воде, а не в масле?»

Почему соль растворяется в воде

Соль или хлорид натрия состоит из ионов натрия и хлорида, соединенных ионной связью с образованием заряженной молекулы NaCl.Вода состоит из двух атомов водорода и одной молекулы кислорода, соединенных ковалентной связью с образованием заряженной молекулы h40. Наибольшее сходство между этими двумя веществами заключается в том, что их молекулы заряжены, что делает их реактивными. Вот почему соль растворяется в воде. Когда соль смешивается с водой, заряженные молекулы воды взаимодействуют с заряженными молекулами соли. Противоположные заряды внутри каждой молекулы раздвигают их, что приводит к растворимости солей в воде. Когда молекулы распадаются, положительные ионы водорода в воде притягиваются к отрицательным ионам хлорида в соли, а отрицательные ионы кислорода притягиваются к положительным ионам натрия.Ионы натрия и хлора равномерно смешиваются с окружающими молекулами воды, и таким образом соль растворяется в воде, образуя гомогенную (равномерно распределенную) смесь.

Magone / iStock / GettyImages

Почему соль не растворяется в масле

Молекулы масла не содержат заряда. Нефть состоит из длинных цепочек атомов водорода и углерода, связанных друг с другом. Он не содержит никаких чистых расходов, что делает его инертным. Итак, соль и масло «химически не похожи».«Один заряжен, другой нет. В результате, когда соль добавляется к маслу, никакие связи не разрываются. Соль и масло просто не смешиваются. При объединении они остаются отчетливо разделенными.

.

Почему вода растворяет соль? | Глава 5: Молекула воды и растворение

Ключевые понятия
  • Полярность молекул воды позволяет воде растворять многие ионно-связанные вещества.
  • Соль (хлорид натрия) состоит из положительных ионов натрия, связанных с отрицательными ионами хлорида.
  • Вода может растворять соль, потому что положительная часть молекул воды притягивает отрицательные ионы хлорида, а отрицательная часть молекул воды притягивает положительные ионы натрия.
  • Количество вещества, которое может растворяться в жидкости (при определенной температуре), называется растворимостью вещества.
  • Растворяемое вещество называется растворенным веществом, а вещество, совершающее растворение, называется растворителем.
Сводка

Студенты сделают двухмерную модель кристалла соли и с помощью вырезок из молекул воды покажут, как вода растворяет соль. Увидев анимацию растворения соли в воде, учащиеся сравнивают, насколько хорошо вода и спирт растворяют соль.Они свяжут свои наблюдения со структурой соли, воды и спирта на молекулярном уровне.

Цель

Студенты смогут объяснить на молекулярном уровне, почему вода может растворять соль. Студенты смогут определить переменные в своем эксперименте. Студенты также смогут объяснить, почему менее полярная жидкость, такая как спирт, не растворяет соль.

Оценка

Загрузите лист активности учащегося и раздайте по одному учащемуся, если это указано в упражнении.Лист упражнений будет служить компонентом «Оценить» каждого плана урока 5-E.

Безопасность

Убедитесь, что вы и ученики носите правильно подогнанные очки. Изопропиловый спирт легко воспламеняется. Держите его подальше от источников огня или искр. Прочтите и соблюдайте все предупреждения на этикетке. Алкоголь следует утилизировать в соответствии с местными правилами.

материалов для каждой группы
  • Строительная бумага любого цвета
  • Ножницы
  • Лента или клей
  • Вода
  • Изопропиловый спирт (70% или выше)
  • Соль
  • Баланс
  • 2 прозрачных пластиковых стакана
  • 2 маленьких пластиковых стакана
  • Цилиндр градуированный
О материалах

Вы можете ламинировать молекулы воды, ионы натрия и ионы хлора, расположенные на последней странице рабочего листа, чтобы вы могли повторно использовать их со своими учениками в следующем году.

.

Суспензии, эмульсии и коллоиды

== >> Для получения дополнительной информации о смесях (растворы, суспензии, эмульсии, Коллоиды )

Суспензии

A суспензия представляет собой смесь двух веществ, одно из которых мелко разделены и рассеяны в другом. Обычные суспензии включают песок в воде, пыль в воздухе и капли масла в воздухе. Частицы в суспензии крупнее чем в решениях; они видны под микроскопом и часто могут быть видно невооруженным глазом.Частицы в суспензии будут оседать, если суспензия разрешено стоять безмятежно. Можно отделить многие частицы суспензии. через фильтр. Примером простой суспензии может быть мука в воде или песок в воде.

Коллоиды

A коллоид — это тип смеси, промежуточной между гомогенной смесью (также называется раствором ) и гетерогенной смесью с свойства также являются промежуточным звеном между ними.

частицы в коллоиде могут быть твердыми, жидкими или пузырьками газа. Среда, которая они во взвешенном состоянии могут быть твердыми, жидкими или газовыми (хотя газовые коллоиды не могут взвешены в газе). Частицы составляют примерно от 10 до 10 000 ангстрем в размер и, как правило, не может быть отфильтрован или урегулирован простым способом. Коллоиды может быть цветным или полупрозрачным из-за эффекта Тиндаля, который представляет собой рассеивание света частицами в коллоиде. Коллоидные частицы можно увидеть в луче света, такого как пыль в воздухе в «валу» солнечного света.

Броуновский движение можно использовать для различения растворов и коллоидов. Броуновский движение — это случайное движение коллоидных частиц, взвешенных в жидкости или газ, вызванный столкновениями с молекулами окружающей среды. В частицы в растворах и коллоидах находятся в постоянном движении. Однако коллоидные частицы достаточно велики, чтобы их можно было наблюдать, и достаточно малы, чтобы на них все же воздействовали случайные столкновения молекул. Коллоидные частицы сопротивляются быстрому осаждению на дно сосуда из-за броуновского движения.

Эмульсии являются разновидностью коллоидных

эмульсий являются примером коллоидов, состоящих из крошечных частиц, взвешенных в другом несмешивающемся (несмешиваемый) материал. Эмульсия представляет собой суспензию двух жидкостей, которые обычно не смешивайте вместе. Эти жидкости, которые не смешиваются, называются несмешивающимися . Примером могут служить масло и вода. Если смешать масло и воду и встряхнуть их, образуется суспензия. Дайте смеси остыть, и масло и вода разделятся.

Ан эмульгатор (эмульгатор) — любое вещество, удерживающее части эмульсия, смешанная вместе. Например, если мы смешаем масло и воду, суспензия будет форма, которая со временем отделяется. Но теперь, если мы добавим несколько капель и встряхнем смесь масло и вода будут оставаться смешанными намного дольше.

Примеры эмульсий включают масло, маргарин и майонез.

См. Также ГИДРОКОЛЛОИДЫ

Примеры коллоидов

Дисперсный Средний
Газ Жидкость цельный
непрерывный Средняя Газ НЕТ
(Все газы растворимы)
Жидкость Аэрозоль
Примеры: туман, туман
Твердое вещество Аэрозоль
Примеры: дым, пыль
Жидкость Пена
Примеры: Взбитые сливки
Эмульсия
Примеры: Молоко, майонез, крем для рук
Sol
Примеры: Краска, пигментные чернила, кровь, магнезиальное молоко
твердое вещество твердое вещество Пена
Примеры: аэрогель, пенополистирол, пемза
Гель
Примеры: Желатин, сыр
Твердый Sol
Примеры: рубиновое стекло
.

Узнайте, что растворяется в воде, с помощью дошкольного научного эксперимента

Всегда есть , что мы можем открыть для себя, а также старые любимые. Я так благодарен за то, что нам вручили задания … Веселые и простые, которые можно объединить в мгновение ока! Ты потрясающий, Джейми, и я благодарен тебе за то, что ты поделился своими делами и идеями !! — Мелисса К.

«Мне очень нравится, что этот берет на себя всю подготовительную работу по привлечению моих детей. Так легко просто повесить календарь и взглянуть на него для вдохновения, когда мы в фанке.»- Участник Activity Room, Рэйчел

Я обнаружил, что невозможно найти в Google идеи с миллиона разных сайтов, организовать, купить товары и т. Д. Это именно то, что я искал! Спасибо, что сделали что-то такое. организован и прост в использовании. — Мелисса К., пользователь планов деятельности в первые годы жизни.

Это избавляет от необходимости рыскать в Интернете в поисках идей. Это все равно что искать рецепт в Интернете, так много вариантов, что часто бывает Не так утомительно смотреть в книгу на полке, чем думать о слишком большом количестве вариантов.- Пользователь планов действий в первые годы, Робин Дж.

Большое спасибо за эту деятельность. Они доказали мне, что Я МОГУ быть той мамой, которая делает крутые и творческие вещи со своими детьми! И эти крутые и креативные вещи могут быть довольно простыми! Какое откровение. Спасибо!! — 7 Day Challenge, Кэти М.

Я чувствую себя молодой мамой, у которой так много забавных идей. Раньше я боялся полудня, после сна, потому что было так скучно делать одно и то же день за днем, но теперь я с нетерпением жду нашего «игрового» времени! — Хейли С.

Вы во многом изменили то, как я провожу время с ребенком! Теперь я один счастливый папа, который больше не задается вопросом, что я собираюсь делать с этим маленьким парнем в течение следующих 12 часов: P Ваш сайт был первым спасателем жизни папы! — Джек С.

Трудно придумать, что бы развлечь и увлечь трехлетнего ребенка, заботясь о ребенке. Все, что мы пробовали на вашем сайте, понравилось трехлетнему ребенку. Ваши идеи настолько просты, что он может воплощать их в жизнь часами.НЕБО! — Карен И.

.

ВОДА – РАСТВОРИТЕЛЬ

ІІ. Актуализация опорных знаний.

Проверка домашнего задания.

Интеллектуальная разминка

Проверяет домашнее задание. Проводит беседу о проделанной работе.

— Определите вещество по его свойствам:

• Непрозрачный, белый, рыхлый, безвкусный, быстро тает в тепле. (Снег.)

• Непрозрачный, белый, твердый, сладкий, быстро тает в воде. (Сахар.)

• Прозрачный, бесцветный, твердый, скользкий, хрупкий, в тепле тает медленно. (Лед.)

• Прозрачное, бесцветное, твердое, скользкое, хрупкое, не тает. (Стекло.)

Отвечают на вопросы учителя. Рассказывают о выполненной работе дома.

Выделять существенную информацию из текста загадки.

Осуществлять актуализацию личного жизненного опыта

IV. Открытие нового знания, способа действия.

Работа по учебнику. Практическая работа

Сидит на одной из парт, с разрешения председателя делает обучающимся первое сообщение:

— Такое же заседание недавно провели ученики села Мирного. Открыл заседание Костя Погодин, который напомнил всем присутствующим еще об одном удивительном свойстве воды: многие вещества в воде могут распадаться на невидимые мельчайшие частицы, то есть растворяться. Следовательно, для многих веществ вода- хороший растворитель. После этого Маша предложила провести опыты и выявить способы, с помощью которых можно будет получить ответ на вопрос, растворяется вещество в воде или нет.

Предлагает председателю выбрать ученика, который прочитает вслух из учебника (с. 85) порядок действий при проведении первого опыта.

— Есть такая сказка. Два ослика шли по дороге с кладью. Один был навьючен солью, а другой — ватой. Первый осел едва передвигал ноги: так тяжела была его ноша. Второй — шел весело и легко.

Вскоре животным пришлось переходить речку. Осел, навьюченный солью, остановился в воде и стал купаться: он то ложился в воду, то снова становился на ноги. Когда осел вышел из воды, ноша его стала гораздо легче. Другой осел, глядя на первого, тоже стал купаться. Но чем дольше он купался, тем тяжелее становилась навьюченная на него вата.

— Почему же ноша первого осла после купания стала легче, а второго — тяжелее?

— Что произошло бы, если б второй осел нес не вату, а сахар?

— Сегодня мы узнаем о возможностях воды как растворителя. В этом нам помогут опыты.

Опыт 1.

— Возьмем стакан с водой. Насыплем соль и размешаем ее. Что происходит с кристаллами соли?

— Но исчезла ли соль?

— Приступим к выполнению другого опыта, для которого необходимо изготовить фильтры.

— Возьмем пустой чистый стакан, бумагу для фильтрования. Ее можно заменить бумажной салфеткой. Салфетку сложим вчетверо, вложим фильтр в воронку, а воронку поставим в пустой стакан. Выльем содержимое стакана в воронку. Что произошло?

— Что происходит с солью в воде?

— Рассмотрите рисунок, на котором показано, как из промокательной бумаги или бумажной салфетки сделать фильтр. Обратите внимание на следующие моменты:

1) сначала надо вырезать из бумаги квадратик, сложить его пополам, затем еще раз пополам;

2) обрезать края так, чтобы при развертывании получился круг;

3) сложить листок в виде конуса, при этом одна половина конуса будет состоять из одного слоя бумаги, другая — из трех слоев;

4) положить фильтр в воронку;

5) смочить правильно вложенный фильтр водой с помощью пипетки.

Опыт 2.

— Возьмем стакан с водой и добавим в него несколько капель вишневого сока. Что произошло с водой?

— Пропустим окрашенную воду через фильтр. Что произошло?

Опыт 3.

— Возьмем стакан с водой и добавим речной песок. Что произошло?

— Пропустим воду через фильтр. Что произошло?

— Что можно сказать о речном песке?

Опыт 4.

— Возьмем стакан с водой и добавим глину. Что стало с водой?

— Давайте немного подождем. Что происходит?

— Пропускаем воду из этого стакана через фильтр. Что получилось?

— Какой можно сделать вывод?

— Что растворяется в воде? Что не растворяется?

Заседание начинается с выбора председателя, который будет его вести. Председатель занимает место за учительским столом. Он объявляет тему заседания (читает ее на открытой перед ним странице учебника), делит обучающихся на четыре группы и рассказывает о заданиях, которые будут выполнять группы.

— Соль растворилась в воде, а вата от воды стала тяжелее.

— Сахар тоже растворился бы в воде.

Выполняют опыты, руководствуясь инструкционными картами. Формулируют выводы наблюдений.

— Кристаллы соли становятся все меньше и меньше и скоро совсем исчезнут.

— Но соль не исчезла. Она растворилась.

Председатель зачитывает вопросы, предшествующие опыту, и каждая группа отвечает на эти вопросы. Например: “Изменилась ли прозрачность воды?” Представитель первой группы выходит и отвечает на вопрос. Председатель еще раз зачитывает тот же вопрос. Выходит, представитель второй группы и отвечает. И так до тех пор, пока не ответит представитель последний группы. Председатель зачитывает второй вопрос: “Изменился ли цвет воды?”. Затем третий: “Изменился ли вкус воды?”.

И наконец, четвертый: “Можно ли сказать, что соль исчезла?”.

— На фильтре ничего не оседает, а вода остается соленой.

— Соль в воде растворяется.

После этого председатель просит всех приступить к выполнению опыта, для которого необходимо изготовить фильтры.

После изготовления фильтра обучающиеся пропускают через него воду с солью, с сахарным песком, затем речным песком и делают вывод: с помощью фильтра нельзя очистить воду от веществ, которые в ней растворились.

— Но фильтр помогает очистить воду от частиц, которые в ней не растворяются.

— Вода окрасилась.

— На фильтре ничего не осело, а вода осталась окрашенной.

— Песчинки падают на дно стакана и лежат там, не изменяясь.

— Вода прошла в стакан, а песок остался на фильтре.

— Песок не растворяется в воде.

— Вода стала мутной, а кусочки глины плавают в воде.

— Частицы глины оседают на дно.

— Вода стала чистой и прозрачной, а частицы глины остались на фильтре.

— Глина не растворяется в воде.

— Сахар, сок, соль растворяются в воде. Песок, глина не растворяются

Планировать решение учебной задачи: выстраивать алгоритм действий, выбирать действия в соответствии с поставленной задачей.

Воспроизводить по памяти информацию, необходимую для решения учебной задачи, обосновывать выбор.

Применять правила делового сотрудничества.

Развивать чувство доброжелательности, эмоционально-нравственную отзывчивость.

Приводить убедительные доказательства в диалоге, проявлять активность во взаимодействии.

Преобразовывать модели в соответствии с содержанием учебного материала и поставленной учебной целью.

Осуществлять контроль по результату

V. Включение нового в активное использование в сочетании с ранее изученным, освоенным.

Самостоятельная работа

Организует беседу, помогает сделать вывод. Уточняет и расширяет знания обучающихся о значении воды в природе.

— Как очистить воду от нерастворимых веществ?

— Что такое фильтрование?

— Почему родниковая вода чистая?

— Для чего нужна вода?

— Почему в жаркий день, прибежав с улицы, вы первым делом выпиваете стакан воды?

Организует самостоятельную работу в тетради (задание 43, с. 30).

— Прочитайте рассказ в хрестоматии “Вода — растворитель” (с. 46). Ответьте на вопрос: “Почему ученым пока не удалось получить абсолютно чистую воду?”.

— Как люди используют свойство воды растворять некоторые вещества?

Благодарит председателя за активную работу и просит остальных членов клуба (по желанию) познакомить одноклассников с содержанием писем, которые получили из Москвы

Отвечают на вопросы учителя, высказывают свои мнения и предположения. Доказывают, аргументируют свою точку зрения.

— С помощью фильтра.

Фильтрование — очистка воды от нерастворимых примесей.

— Родниковая вода проходит сквозь толстый слой песка и очищается.

— Все живые организмы не могут жить без воды.

— Мы пополняем потерянную влагу.

— В воде растворяются не только твердые вещества, но некоторые жидкости (например, уксус, спирт), а также газы, в частности кислород. Благодаря этому в реках, озерах, морях живут рыбы, насекомые, ракообразные, моллюски. Если бы кислород не растворялся в воде, то водоемы были бы безжизненными.

Задание 43: самостоятельно проводят опыты вместе с соседом по парте. Записывают результаты наблюдения.

— Из личного опыта нам, членам клуба, известно, что это свойство человек использует, когда готовит пищу (заваривает чай, варит компоты, супы, солит и консервирует овощи и т. д.). Когда умывается и стирает одежду. Когда рисует акварельными красками или гуашью. Когда насыщает кислородом воду в аквариуме или прорубает зимой лунки во льду в водоемах для улучшения жизни подо льдом. Когда разводит водой лекарства для инъекций (уколов).

В группах обсуждают план выступления на тему “Вода — растворитель”

Выбирать действия всоответствии с поставленной задачей, оценивать уровень владения тем или иным учебным действием, уметь вносить необходимые корректировки в действие после завершения на основе оценки и учёта характера сделанных ошибок.

Осуществлять взаимный контроль, планировать способы взаимодействия

VII. Итог урока.

Рефлексия

Оценка результатов выполнения заданий на уроке.

Организация подведения итогов урока обучающимися.

Предлагает обучающимся оценить свою работу на уроке.

— Какими знаниями, полученными на уроке, вы хотели бы поделиться дома? Какое задание понравилось больше всего? Что вызвало затруднение?

— О каком свойстве воды вы узнали из опытов?

— Какие могут быть вещества?

— Назовите растворимые вещества.

— Назовите нерастворимые вещества.

— Понравилась ли вам работа на уроке? Оцените себя

Отвечают на вопросы. Определяют свое эмоциональное состояние на уроке. Проводят самооценку, рефлексию.

— Вода — хороший растворитель.

— Растворимые и нерастворимые в воде.

— Соль, сахар, мед — растворимые вещества.

— Песок, мел, глина — нерастворимые вещества.

— Сегодня на уроке мы…

Осуществлять итоговый контроль, оценивать результаты деятельности, оценивать уровень владения учебным действием

Разделение смеси соли и речного песка. Химия. 8 класс. Разработка урока

УМК по химии для 8 класса О. С. Габриеляна.

Тип урока: Комбинированный (теория, практическая часть)

Цели урока:

  • Образовательные – ознакомиться с методами получения чистых веществ и способами разделения смесей. Продолжить формирование умений и навыков учащихся по использованию лабораторного оборудования для проведения химического эксперимента.
  • Развивающие – формировать умение наблюдать за явлениями, описывать их и делать умозаключения-выводы. Научить простейшим способам разделения смесей – отстаиванию, фильтрованию, выпариванию.
  • Воспитательные – развивать умения работать индивидуально и в группах, делать практические выводы из произведенного опыта и знаний.

Задачи урока:

  • Обобщить знания по теме «Виды смесей», полученные на предыдущем уроке, путем проведения фронтального опроса в виде беседы.
  • Рассказать о способах разделения смесей, используя ЭОРы.
  • Актуализировать полученную информацию.
  • Объяснить методику выполнения практической части работы.

Оборудование урока: Мультимедийная установка, химические стаканы, воронка, бумажный фильтр, стеклянная палочка, спиртовка, спички, держатель, предметное стекло, смесь соли и речного песка.

Используемые источники: Габриелян О.С. Химия. 8 класс :учеб. Для общеобразоват. учреждений / О.С.Габриелян. – М.: Дрофа, 2012.- 286 с.: ил. Электронное приложение к данному учебнику.

Ход урока

1. Организация класса — 2 минуты.

2. Теоретическая часть урока – 5 минут (беседа)

Учитель: На прошлом уроке мы познакомились с понятием «смеси». Выяснили, что они бывают

  1. Природными.
  2. Искусственными.

Дети:

  1. Морская вода, воздух, гранит.
  2. Бронза, стекло, пластик, различные растворы.

Учитель: На какие две группы смеси делятся по составу?

Дети:

  1. Однородные – визуально не наблюдается граница раздела компонентов (истинные растворы).
  2. Неоднородные – визуально наблюдается граница раздела компонентов смеси (масло и вода).

Учитель: На какие группы делятся смеси по агрегатному состоянию?

Дети:

  1. Газообразные (воздух, пропан-бутановая смесь).
  2. Жидкие (молоко, речная вода, нефть).
  3. Твердые (гранит, бронза, стекло).

Учитель: Для чего человеку нужны чистые вещества?

Дети: Чтобы создавать необходимые смеси нужного состава.

Учитель: Подводя итог моим вопросам и вашим ответам, предположите тему сегодняшнего урока.

Дети сами формулируют тему урока и записывают в тетрадь.

3. Новый материал: «Способы разделения смесей». — 10 минут.

Дистилляция, перегонка, кристаллизация – методы, основанные на разных температурах кипения компонентов смеси. Применяется для однородных смесей (вода, нефть, воздух, растворы) ЭОР №1 (приложение).

Фильтрование – основано на разной пропускной способности фильтров.

Отстаивание – основано на разной плотности веществ.

Фильтрование и отстаивание предназначены для разделения неоднородных смесей (соль и песок, масло и вода) ЭОР №2 (приложение).

Возгонка – это переход твердого вещества в газообразное минуя жидкое. Применяется для получения чистого йода. Встречается в быту, когда мокрое белье высыхает на морозе.

Постановка проблемы.

Учитель: Вы пошли в лес на пикник, запас соли рассыпался на землю. Что вы будете делать?

Дети: Звучат разные варианты ответов.

4. Практическая часть урока «Разделение смеси соли и речного песка». — 20 минут.

Дети получают методички, оборудование. Перед началом выполнения работы изучают алгоритм своих практических действий. ЭОР №3 (приложение).

Цель работы: практически провести разделение смеси.

Оборудование: химический стакан (2 шт.), воронка, фильтр, спиртовка, спички, держатель, стекло для выпаривания, смесь речного песка с солью.

Ход работы

Задание №1: выделить поваренную соль из смеси с песком.

  1. Растворить смесь песка и соли в воде.
  2. Приготовьте фильтр и вложите его в воронку.
  3. Отфильтруйте смесь.
  4. Поместите несколько капель фильтрата на стекло для выпаривания, закрепите стекло в держателе и нагрейте над пламенем спиртовки до появления кристаллов поваренной соли.

Задание №2

Сделайте рисунки ваших действий.

Задание №3: Ответить на вопросы.

  1. Каков характер разделяемой смеси (однородная или неоднородная)
  2. На чем основаны методы разделения смеси.
  3. Учебник стр.155. Вопросы № 3, 4, 5, 6.

5. Заключительная часть урока — 8 минут.

Отвечают устно на вопросы (учебник стр.155. №3,5,6).

Смотрят ЭОР №4 (приложение) «Как профильтровать воду в походных условиях»

Отделение песка и соли фильтрованием и испарением | Эксперимент

Это очень простой эксперимент. Его можно проводить индивидуально или в группах по два человека. Ученики должны вставать во время обогрева и опасаться выплевывания горячей соли, когда испарение почти завершится.

Оборудование

Аппарат

  • Защита глаз
  • Стакан, 250 см 3
  • Стеклянный стержень для перемешивания
  • Фильтровальная воронка
  • Фильтровальная бумага
  • Колба коническая, 250 см 3
  • Испарительная ванна
  • Горелка Бунзена
  • Термостойкий мат
  • Штатив
  • Марля

Химическая промышленность

  • Смесь песка и хлорида натрия (соль), около 6–7 г на группу учащихся (подходящая песчано-солевая смесь должна содержать примерно 20% соли по массе)

Примечания по безопасности, охране труда и технике

  • Во время эксперимента пользуйтесь защитными очками.
  • Ученики должны вставать во время обогрева и опасаться выплевывания горячей соли, когда испарение почти полностью.
  • Хлорид натрия (например, поваренная соль), NaCl (и) — см. CLEAPSS Hazcard HC047b.

Процедура

  1. Залейте песчано-солевую смесь в стакан так, чтобы она покрывала дно.
  2. Добавьте примерно 50 см. 3 воды или добавьте воды, пока стакан не будет заполнен примерно на одну пятую.
  3. Осторожно перемешайте смесь в течение нескольких минут.
  4. Отфильтровать смесь в коническую колбу.
  5. Перелейте фильтрат в испарительную ванну.
  6. Осторожно нагрейте солевой раствор до тех пор, пока он не начнет стекать (плевать). УХОД: Наденьте защиту для глаз и не подходите слишком близко.
  7. Выключите конфорку Бунзена и дайте влажной соли высохнуть в посуде.
Показать полноэкранный режим

Учебные заметки

При желании эксперимент можно расширить для выделения сухих образцов песка и соли.Для этого влажный песок в фильтровальной бумаге можно перенести на другой лист сухой фильтровальной бумаги, и, сложив и промокнув, образец можно высушить. При необходимости можно использовать другой кусок фильтровальной бумаги.

Студенты часто любят представлять свои образцы в маленьких бутылочках для утверждения, поэтому для этого можно использовать шпатель. Пока первый ученик в паре переносит песок, другой может соскребать засохшую соль с чашки для испарения и переносить ее в другую бутылку с образцами.

Если это расширение проводится, учеников следует поощрять маркировать бутылки. Им следует сказать, что все образцы, приготовленные таким образом, должны быть маркированы, даже если в этом случае должно быть очевидно, какое вещество какое.

Вопросы студентов
  1. Почему с помощью этого эксперимента можно разделить песок и соль?
  2. Почему смесь соли, песка и воды перемешивается на шаге 3?
  3. Почему солевой раствор нагревается на шаге 6?
  4. Как можно окончательно удалить следы воды из ваших образцов, чтобы они полностью высохли?
  5. Назовите две причины, по которым добытый вами песок все еще может быть загрязнен солью.
  6. Как бы вы могли адаптировать свой эксперимент, чтобы получить более чистый образец песка?
  7. Назовите две причины, по которым полученная вами соль все еще может быть загрязнена песком.
  8. Как бы вы могли адаптировать свой эксперимент для получения более чистого образца соли?

Учебные материалы по начальным наукам

Если вы преподаете основы естествознания, следующая информация призвана помочь вам в использовании этого ресурса.

Развитие навыков

Дети будут развивать свои научные рабочие навыки по:

  • Делаем выводы и поднимаем дополнительные вопросы, которые можно было бы исследовать, на основе их данных и наблюдений.
  • Использование соответствующего научного языка и идей для объяснения, оценки и передачи своих методов и результатов.

Результаты обучения

Детей будет:

  • Обратите внимание на то, что некоторые материалы растворяются в жидкости с образованием раствора.
  • Опишите, как извлечь вещество из раствора.
  • Используйте знания о твердых телах, жидкостях и газах, чтобы решить, как можно разделить смеси, в том числе посредством фильтрации, просеивания и испарения.
  • Продемонстрируйте, что растворение, перемешивание и изменение состояния являются обратимыми изменениями.

Поддерживаемые концепции

Детей выучат:

  • Что существуют различные методы, которые можно использовать для разделения различных смесей.
  • Это растворение — обратимая реакция.
  • Что не все твердые вещества растворимы.
  • Что на скорость растворения могут влиять различные факторы.
  • Что плавление и растворение — не одно и то же.

Предлагаемое использование деятельности

Это задание можно использовать как расследование для всего класса, когда дети работают в небольших группах или парах, чтобы понять, как отделить соль от песка. Это может послужить стимулом для дальнейших исследований, направленных на то, как разделить другие смеси твердых веществ либо с разными размерами частиц, либо по растворимости.

Практические соображения

В начальных школах часто нет горелок Бунзена, поэтому необходимо найти жизнеспособные альтернативы.Точно так же может быть трудно найти оборудование, необходимое для испарения воды с целью извлечения растворенной соли. Подголовники и чайные свечи могут подойти как возможные альтернативы.

При выполнении этой деятельности имейте в виду, что некоторые нерастворимые твердые вещества могут образовывать суспензии. Кажется, что именно здесь частицы растворились, хотя на самом деле они распространились по всей жидкости. Хорошим индикатором того, что образовалась суспензия, является то, что жидкость станет мутной или будет слышен скрежет частиц при перемешивании смеси.

Схема этого упражнения очень предписывающая, поскольку процедура изложена поэтапно. Открытые задания, в которых дети работают в небольших группах и придумывают собственные методы, расширили бы мышление детей. Предложения разных групп можно было сравнивать и оценивать как класс.

Дополнительная информация

Это ресурс из проекта «Практическая химия», разработанного Фондом Наффилда и Королевским химическим обществом.Этот сборник из более чем 200 практических занятий демонстрирует широкий спектр химических концепций и процессов. Каждое упражнение содержит исчерпывающую информацию для учителей и технических специалистов, включая полные технические заметки и пошаговые инструкции. Практическая химия сопровождает практическую физику и практическую биологию.

© Фонд Наффилда и Королевское химическое общество

Проверено на здоровье и безопасность, 2016

Разделительные смеси | S-cool, сайт доработки

Для отделения твердого вещества от жидкости обычно используются четыре метода:

1. Фильтрация

2. Испарение

3. Кристаллизация

4. Центрифугирование

Фильтрация

Если твердое вещество не растворяется в воде, например мел, то вы можете отделить твердое вещество от жидкости, отфильтровав суспензию через фильтровальную бумагу.

В этом примере мел, оставшийся на фильтровальной бумаге, называется остатком. Вода (вещество), которая проходит через фильтровальную бумагу, называется фильтратом .

Испаритель

Если ваша смесь представляет собой раствор, например соленую воду, то при фильтрации соль не отделится от воды.

Вместо этого при нагревании растворенного вещества растворитель (вода) испаряется, оставляя твердое вещество (соль). Это возможно, поскольку эти два вещества имеют разные точки кипения.

Центрифугирование

Центрифуга используется для отделения небольших количеств твердой фазы, содержащейся в суспензии, от жидкости.Например, мел от воды.

Центрифуга содержит пробирки, которые вращаются с высокой скоростью, в результате чего твердое вещество опускается на дно пробирки. Жидкость декантируется (сливается), оставляя твердое вещество.

Кристаллизация

Можно разделить множество твердых веществ, содержащихся в насыщенных растворах , оставив их для образования кристаллов. Этот процесс называется кристаллизацией .

Если раствор является насыщенным, то когда растворитель испаряется, то, что остается, не может удерживать столько растворенного вещества.Таким образом растворенное вещество покидает раствор и образует кристаллы.

Путем растворения одного из двух твердых веществ

Если у вас есть смесь соли и песка, то, поместив смесь в воду, вы обнаружите, что соль растворяет , а песок остается.

Если эту новую смесь затем фильтровать , соль в растворе соленой воды проходит через фильтровальную бумагу с образованием фильтрата, а песок остается в виде остатка .

Все, что теперь осталось сделать, — это нагреть соленую воду, чтобы она испарилась, оставив соль.

Как отделить растворитель от раствора

Простая перегонка

Например: для получения чистой воды из соленой воды можно использовать это устройство.

Раствор нагревают в круглодонной колбе. По мере кипения пар поднимается в конденсатор (он снова охлаждает пар до воды).В конце концов соль (растворенное вещество) остается. Вода, собранная в химическом стакане, называется дистиллированной водой .

Фракционная перегонка

Если две жидкости смешиваются (т.е. они хорошо смешиваются), их можно разделить с помощью этого устройства.

Например: для разделения смеси этанола и воды.

Так как этанол кипит при 78 o C, а вода при 100 o C, затем при постепенном нагревании смеси этанол и водяной пар поднимаются вверх по колонке, нагревая стеклянные шарики по мере их конденсации на них.

Когда температура шариков составляет 78 o ° C, пары этанола нагнетаются в конденсатор, в то время как водяной пар продолжает конденсироваться и капать обратно в колбу. Это означает, что этанол, находящийся сейчас в конденсаторе, конденсируется и капает в химический стакан в виде жидкого этанола.

Воронка делительная

Если две жидкости не смешиваются, используется делительная воронка.

Например: Если вы наливаете смесь масла и воды в воронку, масло всплывает на поверхности воды.Все, что остается сделать, это открыть кран, чтобы вода могла вытечь. Кран закрывается после того, как вытечет вся вода.

Бумажная хроматография

Например: для разделения цветных веществ черными чернилами — можно использовать устройство, указанное ниже.

1. Небольшая капля черных чернил помещается в центр фильтровальной бумаги.

2. Затем на чернила капают воду.

3. Чернила медленно растекаются, разделяясь на кольца разного цвета.

4. Фильтровальная бумага с цветными кольцами называется хроматограммой.

5. Цветное вещество, наиболее удаленное от исходного пятна черных чернил, является наиболее растворимым.

Как разделить песок и соль

Разделите песок и соль, используя разницу в растворимости, плотности или температуре плавления. (фото: Castorly)

Вы можете разделить песок и соль либо для получения компонентов, либо для изучения химии разделения.Разделение — это метод очистки, основанный на физических или химических различиях между двумя или более материалами. Вот четыре способа отделить песок от соли и взглянуть на используемые принципы. Это отличный научный проект для учащихся любого уровня обучения, потому что он заставляет их задуматься о свойствах материи, используя знакомые материалы. Они могут применить научный метод для тестирования различных методов разделения.

Разделение песка и соли механически

Компоненты любой смеси, такие как песок и соль, разделяются на основе их различий.И песок, и соль представляют собой твердые частицы с частицами одинакового размера. Однако при увеличении частицы выглядят иначе. Один из способов отделить песок от соли — использовать пинцет для механического отделения зерен по частям. Это не эффективный метод разделения, но он работает.

Разделение песка и соли по плотности

Вы можете гораздо быстрее разделить песок и соль, используя различную плотность двух веществ. Плотность соли составляет 2,16 г / см³, а плотность песка — 2.65 г / см³. Иными словами, если вы наполните ведро солью, а другое — песком, то ведро с песком будет весить больше. Песок немного тяжелее соли.

Итак, если встряхнуть сковороду с песком и солью, соль поднимется до верха сковороды. Вы можете соскрести верхний слой соли, чтобы отделить его от песка. По такому же принципу мы собираем золото. Золото тяжелее большинства других веществ, поэтому остается на дне кастрюли.

Разделение песка и соли по растворимости

Эти женщины отделяют морскую соль от песка и воды, позволяя солнцу испарить воду.(фото: Куанг Нгуен Винь)

Соль растворяется в воде, а песок — нет. Другими словами, соль растворима в воде, а песок — нет. Итак, вы можете использовать разницу в растворимости для разделения смеси песка и соли. Добавляйте в смесь воду до растворения соли. Растворимость увеличивается с повышением температуры, поэтому, если она горячая, вам понадобится меньше воды. Как только соль растворится, можно собирать песок. Один из способов сделать это — пропустить песок и соленую воду через кофейный фильтр или мелкое ситечко. Вы можете восстановить соль, кипятя соленую воду, пока вся вода не испарится.Кроме того, вы можете просто налить соленую воду в кастрюлю и дать воде медленно испариться. Люди получают морскую соль, разливая морскую воду на большой площади и позволяя солнцу испарить воду.

Разделение песка и соли по температуре плавления

Соль (хлорид натрия) и песок (диоксид кремния) имеют разные точки плавления. Температура плавления соли составляет 1474 ° F (801 ° C), а у песка — 3110 ° F (1710 ° C). Итак, если приложить тепло, соль тает раньше, чем песок. Вы можете использовать разницу температур плавления для разделения смеси песка и соли, нагревая смесь выше 801 ° C, но ниже 1710 ° C.Слейте расплавленную соль, оставив песок. Хотя можно разделить компоненты смеси по температуре плавления, это не очень практично. Это связано с высокими температурами, и песок будет загрязнен небольшим количеством соли. Когда вы сливаете жидкую соль, небольшое количество соли загрязняет песок, например, когда вы сливаете воду с песка.

Вопросы и ответы для студентов

Вот несколько вопросов, которые нужно задать студентам, чтобы они задумались о том, как работает разделение:

  • Какой метод лучше всего подходит для разделения песка и соли? Ответ: Лучше всего работает растворимость, потому что в соленой воде не остается песка, а на песке остается очень мало соли.Если промыть оставшийся песок водой, его можно вскипятить, чтобы получить оставшуюся соль.
  • Извлекает ли из смеси 100% соли и песка каким-либо методом? Ответ: Не совсем. Хотя это происходит медленно, сбор отдельных частиц песка и соли, вероятно, дает лучшее разделение. Однако на молекулярном уровне, вероятно, на поверхности песка есть немного соли. Очищение всегда подразумевает небольшую потерю.
  • Как сделать разделение с использованием растворимости более эффективным? Ответ: Используйте кипящую воду, чтобы растворить соль, чтобы минимизировать количество необходимой воды.Чтобы извлечь соль, налейте соленую воду тонким слоем, чтобы увеличить площадь поверхности и ускорить испарение.
  • Почему, используя метод определения растворимости, соль не испаряется с водой? Ответ: Вода имеет гораздо более высокое давление пара, чем соль. Вода также имеет гораздо более низкую температуру кипения, чем соль. Использование различных точек кипения является основой метода разделения, называемого дистилляцией.
  • Сработает ли какой-либо из этих методов для разделения соли и сахара? Ответ: Вы можете разделить их пинцетом при увеличении (хотя вы бы ненавидели свою жизнь).Кристаллы соли имеют кубическую форму, а кристаллы сахара — гексагональные призмы. Вы можете использовать растворимость. Сахар растворяется в спирте, а соль — нет. Хотя эти два соединения имеют разные точки кипения, вам нужно быть осторожным, чтобы не поджечь сахар.

Список литературы

  • Национальные академии наук, инженерии и медицины (2019). Программа исследований для преобразования науки о разделении (отчет). Вашингтон, округ Колумбия: The National Academies Press. DOI: 10.17226 / 25421
  • Уилсон, Ян Д.; Адлард, Эдвард Р .; Кук, Майкл; и др., ред. (2000). Энциклопедия науки о разделении . Сан-Диего: Academic Press. ISBN 978-0-12-226770-3.

Похожие сообщения

Глава 3: Разделение смесей — веб-сайт г-на Хельмера

3.2: Разделение механических смесей

1. Пример ответа: четыре метода разделения механической смеси: сортировка, отстаивание, фильтрация и растворение.
2.

a) Металлы на свалке металлолома можно разделить путем сортировки.
б) Соль и песок можно разделить, смешав эти два вещества с водой. Соль растворяется в воде, а песок оседает на дне смеси.
c) Песок и гравий можно отделить, вылив смесь в сито с достаточно маленькими отверстиями, чтобы улавливать гравий, но достаточно большими, чтобы пропустить песок.
d) Песок и воду можно отделить с помощью фильтра — вода будет проходить через фильтр, а песок — нет.

3. Пример ответа: Фильтры позволяют чистому воздуху проходить через них, но они задерживают крошечные твердые частицы, которые делают воздух вредным для дыхания.

4. Фильтр похож на сито в том, что оба используются для разделения смесей путем улавливания более крупных веществ в отверстиях, через которые проходят более мелкие вещества. Однако фильтр имеет отверстия гораздо меньшего размера, чем сито, поэтому он используется для отделения крошечных нерастворимых твердых частиц от жидкостей или газов. Сита могут отделять более крупные твердые частицы от более мелких твердых частиц или жидкостей.

3.4: Защита окружающей среды путем разделения смесей

1. Два основных способа избежать загрязнения воды — это выявление и ограничение источников загрязнения воды и очистка загрязненной воды до того, как она попадет в реки и озера.


2. Отстаивание используется при первичной очистке сточных вод для отделения твердых отходов от жидких.

3. Процесс очистки сточных вод состоит из трех основных этапов. Во время первичной обработки просеивание, всплывание и осаждение используются для отделения воды от твердых материалов и компонентов, таких как масло и жир. При вторичной обработке бактерии разрушают любые оставшиеся отходы жизнедеятельности человека и растения. При окончательной (третичной) обработке любой азот или фосфор из воды удаляют путем фильтрации, воздействия ультрафиолетового излучения или озона или использования бактерий.Затем воду обрабатывают хлором, чтобы избавиться от оставшихся организмов.


4. Загрязняющие вещества могут попасть в воду через заводские отходы. Пестициды, соли и удобрения с ферм и дорог также загрязняют воду. Загрязняющие вещества также попадают в воду в результате случайных разливов или утечек.

5. После смешивания очень трудно разделить масло и воду, потому что они оба являются жидкостями. В результате многие методы разделения не очень эффективны.

6.Пример ответа: Разливы нефти могут поставить под угрозу дикую природу, загрязняя океаны, разрушая дома животных, которые там живут. Масло также может отравить дикую природу.

3.5: Разделение растворов


1. Сахар можно отделить от смеси сахара и воды путем кипячения смеси до тех пор, пока вода не испарится, оставив сахар.

2. Пример ответа: В процессе перегонки смесь жидкостей нагревается (метка A на схеме ниже) до тех пор, пока одна из жидкостей не достигнет точки кипения и не испарится.Газ поднимается и направляется в конденсатор (B). Холодная вода в конденсаторе охлаждает газ до тех пор, пока он снова не станет жидкостью (C), а затем эта жидкость собирается в отдельном контейнере (D).


3.a) Кленовый сироп получают выпариванием. Кленовый сок кипятят, чтобы удалить воду и сконцентрировать сахар и другие вещества.
б) При кипячении кленового сока вода выходит из смеси. Вреден ли этот процесс для окружающей среды, зависит от того, как вы нагреваете сок.Вода не наносит вреда окружающей среде, но если для нагрева смеси используются определенные виды топлива (например, природный газ или древесина), могут выделяться газообразные загрязнители.

3.7: Смесь в промышленности

2. a) Зерна пшеницы, смешанные с косточками и крупными палками, можно отделить с помощью сита.
б) Зерна пшеницы, смешанные с кусками металла, можно отделить с помощью магнита.
c) Смесь различных компонентов масла, которые испаряются при разных температурах, можно разделить в процессе дистилляции.

3. При производстве муки используются несколько методов разделения смесей. Сетки отфильтровывают зерна пшеницы от палочек и камней. Вакуумные экстракторы удаляют более легкие загрязнения, например, пыль. Магниты удаляют частицы железа и стали, смешанные с зернами.

4

а) Уран можно использовать для производства электроэнергии, поскольку он является радиоактивным веществом. Когда он разрушается, он высвобождает энергию, которую можно использовать для производства электроэнергии.
б) Электроэнергия может быть произведена путем добавления урана в активную зону ядерного реактора.Когда уран распадается и выделяет энергию, он нагревает воду, окружающую активную зону. Вода закипает и испаряется, а образующийся пар вращает лопасти турбин, вырабатывающих электричество.

5. Уран получают из урановой руды, своего рода горной породы, путем измельчения руды и добавления раствора, растворяющего уран. Затем раствор просеивают и упаривают, оставляя после себя твердый уран.

6.
a) С урановой рудой нельзя обращаться так же, как с обычным мусором, потому что она очень радиоактивна и загрязнила бы почву и воду, если бы ее закопали на свалке.
б) Образец ответа: Иногда урановую руду утилизируют, смешивая ее с расплавленным стеклом и закапывая смесь глубоко в неиспользуемую шахту.

7. Пример ответа: Атомная энергетика в некотором роде очень рискованна. Несчастный случай может привести к утечке радиации, которая может вызвать смертельные заболевания, такие как рак. Кроме того, ядерные отходы очень вредны для окружающей среды. Однако у ядерной энергетики есть и определенные преимущества. Например, он не производит загрязнения, приводящего к кислотным дождям, и не выделяет углекислый газ, который усугубляет проблему изменения климата.

Декантация, загрузка, фильтрация | Класс 6, Разделение веществ

Вопрос 1 Что такое седиментация?

Вопрос 2 Что такое декантация?

Вопрос 3 Как вы разделите смесь песка и воды? Объясните с помощью схемы?

Вопрос 4 Какое вещество используется для загрузки частиц глины, взвешенных в мутной воде?

Вопрос 5 Определите термин «осадок»?

Вопрос 6 Как вы отделите рис и бобовые от воды после мытья?

Вопрос 7 Как вы разделите смесь масла и воды?

Вопрос 8 Что такое фильтрация?

Вопрос 9 Определите термины фильтровальная бумага, остаток и фильтрат?

Вопрос 10 Как вы отделите смесь мела от воды?

Вопрос 11 Как вы разделите смесь песка и воды?

Вопрос 12 Объясните процесс отделения чайных листьев от приготовленного чая?

Также прочтите Решения NCERT для главы 5 Разделение веществ

Седиментация

Твердое вещество, которое оседает на дне жидкости, называется осадком .Осаждение твердого вещества на дне жидкости называется осаждением .

После осаждения получаем два слоя. Верхний слой представляет собой прозрачную жидкость, а нижний слой — осадок.

Декантация

Выливание жидкости из сосуда без нарушения осадка называется декантацией .

Есть смеси, которые содержат нерастворимые твердые частицы, взвешенные в жидкости. Твердые частицы, нерастворимые в жидкости, можно отделить декантацией.

Смесь песка и воды содержит частицы песка, взвешенные в воде. Песок можно отделить от песчано-водяной смеси методом декантации.

В стакан набирают смесь песка и воды. Он содержит частицы песка, разбросанные по воде. Дайте смеси песка и воды постоять в течение некоторого времени. При хранении более тяжелые частицы песка оседают на дне стакана. Этот слой песка называется осадком . Отложение песка на дне стакана называется осаждением .Когда песок осядет, над слоем песка останется чистая вода.

С помощью стеклянной палочки аккуратно переливаем чистую воду в другой стакан, не нарушая отложения песка. Этот процесс называется декантацией . Когда вся чистая вода слита, на дне стакана остается только песок. Таким образом, смесь песка и воды была разделена на песок и чистую воду путем декантации.

Процесс декантации используется в повседневной жизни

Рис и бобовые, купленные на рынке, обычно содержат частицы пыли или почвы в качестве примесей.Итак, рис и бобовые всегда моются перед приготовлением. Когда мы добавляем воду в рис или бобовые, взятые в сосуд, такие примеси, как пыль и частицы почвы, отделяются и уходят в воду. Из-за этого вода становится немного грязной. Рис или бобовые, будучи тяжелыми, остаются на дне емкости, а грязная вода отделяется декантацией путем наклона емкости в сторону.

Процесс декантации можно использовать для разделения только тогда, когда твердое вещество не растворяется в жидкости .

Сахар растворяется в воде, поэтому смесь сахара и воды невозможно разделить методом декантации.

Метод декантации можно также использовать для разделения двух несмешивающихся жидкостей.

Вода и масло — две несмешивающиеся жидкости, которые можно разделить декантацией. Смесь воды и масла выдерживают в химическом стакане. Поскольку вода и масло не смешиваются, они образуют в стакане два отдельных слоя. Вода, будучи более тяжелой, образует нижний слой, а масло, более легкое, образует верхний слой в стакане.Мы можем аккуратно перелить верхний слой масла в другой стакан. Вода останется позади.

Процесс декантации нельзя использовать для разделения двух смешивающихся жидкостей.

Например: спирт и вода смешиваются друг с другом, смесь спирта и воды нельзя разделить декантацией.

Керосин и бензин нельзя разделить декантацией, поскольку они являются смешивающимися жидкостями.

Загрузка

Иногда вода из реки, озера или пруда бывает мутной из-за присутствия в ней взвешенных мелких частиц глины.Эти взвешенные частицы глины очень маленькие, и им нужно много времени, чтобы осесть самостоятельно. Оседание взвешенных частиц глины или мутной воды может быть ускорено процессом загрузки.

Загрузка — это процесс, при котором частицы квасцов осаждаются на взвешенных частицах глины в мутной воде, что делает их тяжелыми и быстро оседает.

Мутная вода представляет собой смесь воды и взвешенных частиц глины. Мутную воду набирают в стакан. Берется большой кусок квасцов и привязывается к нему ниткой.Затем кусок квасцов помещают в стакан с мутной водой и вращают в нем три или четыре раза, удерживая за нить. После этого из стакана вынимается кусок квасцов. Когда кусок квасцов вращается в мутной воде, часть квасцов растворяется в воде. Частица растворенных квасцов осаждается на взвешенных частицах глины мутной воды. Таким образом, взвешенные частицы глины наполняются тяжелыми частицами квасцов. Из-за этого частицы глины становятся довольно тяжелыми и довольно быстро оседают на дне стакана.Когда все взвешенные частицы глины осядут, вода станет прозрачной. Затем чистую воду переливают в другой стакан. Загруженные частицы глины остаются в первом стакане.

Фильтрация

Фильтровальная бумага представляет собой круглый кусок специальной бумаги с миллионами крошечных отверстий. Отверстия в фильтровальной бумаге настолько малы, что их невозможно увидеть невооруженным глазом. Жидкости могут проходить через крошечные отверстия в бумаге, но большие твердые частицы не могут проходить через крошечные отверстия фильтровальной бумаги.

Твердое вещество, которое остается на фильтровальной бумаге, называется остатком .

Жидкость, которая проходит через фильтровальную бумагу, называется фильтратом .

Прежде чем использовать фильтровальную бумагу в процессе фильтрации, ее должным образом складывают, чтобы получился полый конус.

Процесс удаления нерастворимых твердых частиц из жидкости с помощью фильтровальной бумаги известен как фильтрация . Фильтрация используется для отделения нерастворимых веществ от жидкости. Смесь нерастворимого твердого вещества и жидкости наливают в конус из фильтровальной бумаги, закрепленный в воронке, используя стеклянный стержень в качестве направляющей. Жидкость проходит через фильтровальную бумагу и собирается в стакане под воронкой. Твердые частицы не проходят через фильтровальную бумагу и остаются на фильтровальной бумаге. Смесь нерастворимого твердого вещества в жидкости разделяют на твердую и прозрачную жидкость.

Смесь мела и воды отделяется фильтрованием . Когда смесь мела и воды выливается на фильтровальную бумагу, закрепленную в воронке, чистая вода проходит через фильтровальную бумагу и собирается в виде фильтрата.Частицы мела остаются на фильтровальной бумаге в виде осадка.

Смесь песка и воды также может быть отделена фильтрованием . Когда смесь песка и воды выливается на фильтровальную бумагу, закрепленную в воронке, чистая вода проходит через фильтровальную бумагу и собирается в виде фильтрата. Песок остается на фильтровальной бумаге в виде осадка.

В быту используются разные фильтры. например проволочная сетка, кусок хлопка, кусок муслиновой ткани, ситечко для чая.

Чтобы отделить использованные чайные листья, заливаем приготовленный чай через ситечко. Ситечко для чая имеет проволочную сетку, которая действует как фильтр. Жидкий чай проходит через маленькие отверстия ситечка для чая и собирается в чашке внизу. Чайные листья не проходят через ситечко, а чайные листья остаются на ситечке. использованных чайных листьев отделяются от приготовленного чая методом фильтрации.

Фруктовый сок также фильтруется с помощью ситечка в качестве фильтра для удаления семян и твердых частиц мякоти.

Когда молоко охлаждается после закипания, на его поверхности образуется слой сливок. Этот крем можно удалить из молока, профильтровав его через кусок ткани. Молоко проходит через одежду, а сливки остаются на ткани.

На водоочистных сооружениях речная или озерная вода фильтруется с помощью песчаных фильтров .

Смесь двух жидкостей нельзя разделить фильтрацией. Смесь молока и воды нельзя отделить фильтрованием.

Химический эксперимент по извлечению соли из смеси | Education

Химические смеси включают два или более элемента, которые химически не соединены, но существуют в одном и том же физическом месте. Свойства элементов определяют, как они могут быть разделены, например, является ли какой-либо элемент растворимым, обладает ли магнитной восприимчивостью или разделен по цвету, вкусу или запаху. Соль растворима, что позволяет легко отделить песчано-солевую смесь.

Подготовка к эксперименту

Соберите стеклянный стакан, воду, палочку для перемешивания, чашу для испарения, воронку, фильтровальную бумагу, бунзеновскую или спиртовую горелку, спички, металлическую сетку, металлическую подставку для готовки, стеклянную крышку испарителя, щипцы, рукавицы для духовки и ваша смесь соли и песка. Чтобы определить процентное содержание соли в песке, используйте весы, бумагу и письменные принадлежности, чтобы взвесить и записать вес пустого стакана и общий вес чашки для испарения, крышки и палочки для перемешивания.

Первый этап

Поместите смесь в стеклянный стакан и добавьте в два раза больше дистиллированной воды по объему, чем у вас есть смесь песка и соли. Энергично перемешивайте смесь в течение пяти минут, чтобы соль полностью растворилась. Дайте стакану постоять, чтобы песок опустился на дно стакана. Когда песок осядет на дно, медленно вылейте воду из стакана через воронку, выстланную фильтровальной бумагой, в чашу для испарения, чтобы удалить из воды оставшийся песок.

Опреснение воды

Установите чашу для испарения поверх металлической сетки на кухонной подставке. Включите конфорку и дайте воде закипеть и испариться с посуды. Когда соль начнет лопаться, аккуратно поместите стеклянную крышку на емкость для испарения с помощью щипцов, чтобы соль не вылетела из емкости. Выключите горелку, когда соль перестанет появляться, так как вся вода испарится, оставив кристаллизованную соль. Снимите форму с прихваткой для духовки и дайте ей остыть перед взвешиванием.

Удаление воды из песка

Если вы планируете рассчитать процентное содержание соли в песке, удалите воду из песка, нагревая песок так же, как вы делали это с соленой водой. Используйте меньшее пламя с песком, так как для удаления воды потребуется меньше тепла. Когда песок станет сухим, выключите горелку и дайте стакану полностью остыть перед взвешиванием.

Определение процентного содержания смеси

Чтобы определить процентное содержание соли в песке, взвесьте чашу для испарения, накройте и снова палку для перемешивания и вычтите вес пустых контейнеров из веса контейнеров после кристаллизации соли, чтобы получить вес соли. .Взвесьте стакан с сухим песком и вычтите из этого числа вес пустого стакана, чтобы получить вес песка. Добавьте веса песка и соли. Чтобы определить процентное содержание песка, разделите вес песка на вес смешанной соли и песка и умножьте на 100. Если у вас есть 1,015 грамма песка, которые вы отделили от 3,010 грамма смеси песка и соли, у вас есть смесь. это было 33,72 процента песка.

Ссылки

Автор биографии

Преподобный Кэтрин Рэтелиф Барр преподает роды, воспитание детей, вакцинацию и альтернативную медицину с 1994 года.Она является пастырским семейным консультантом и имеет отцовских родителей, приемных, приемных и приемных детей. Она имеет степень бакалавра по английскому языку и истории от Сентенери Колледж Луизианы. Исследования включают акушерство, натуропатию и другие альтернативные методы лечения.

Решения NCERT для науки 6 класса Глава 5

Страница № 55:
Вопрос 1:

Большинство веществ, которые мы видим вокруг себя, — это
(а) чистые элементы.
(б) чистые соединения.
(c) нечистые элементы.
(г) смеси.

Ответ:

(d) смеси

Большинство веществ, которые мы видим вокруг себя, представляют собой смеси различных элементов и соединений.

Страница № 55:
Вопрос 2:

Раствор соли в воде представляет собой соединение
(а).
(б) однородная смесь.
(в) гетерогенная смесь. Элемент
(d).

Ответ:

(б) гомогенная смесь

Когда соль растворяется в воде, соль равномерно распределяется в растворе. Таким образом, солевой раствор представляет собой однородную смесь.

Страница № 55:
Вопрос 3:

Свойство, используемое при разделении смеси двух твердых частиц путем веялки, — это
(a) разница в цвете.
(б) разница в размере.
(c) разница в весе.
(г) притяжение магнитом.

Ответ:

(c) разница в весе

Провеивание используется для разделения двух твердых частиц разного веса, таких как пшеница и полова.

Страница № 55:
Вопрос 4:

Процесс осаждения частиц твердого вещества в жидкости — это
(а) декантация.
(б) сублимация.
(в) седиментация.
(г) фильтрация.

Ответ:

(c) осаждение

Процесс осаждения твердых частиц в жидкости известен как осаждение.

Страница № 55:
Вопрос 5:

Какой метод используется для получения чистой жидкости из раствора?
(а) дистилляция
(б) конденсация
(в) фильтрация
(г) загрузка

Ответ:

(а) дистилляция

Дистилляция может использоваться для получения чистой жидкости из раствора.

Страница № 56:
Вопрос 6:

Температура насыщенного раствора сахара в воде повышена на 10 ° C. Какое из этих утверждений теперь верно?
(a) Он будет продолжать насыщаться.
(b) Он станет ненасыщенным.
(c) Остается ли он насыщенным или становится ненасыщенным, зависит от количества взятой воды.
(d) Он будет продолжать оставаться насыщенным, но если температура повысится выше 10 ° C, он станет ненасыщенным.

Ответ:

(b) он станет ненасыщенным

Если температура насыщенного раствора сахара увеличится на 10 o C, тогда в растворе может раствориться больше сахара, и раствор станет ненасыщенным.

Страница № 56:
Вопрос 7:

Что из следующего может растворяться в воде?
(a) только твердые вещества
(b) только твердые вещества и жидкости
(c) твердые вещества, жидкости и газы
(d) только жидкости

Ответ:

(c) твердые вещества, жидкости и газы

Твердые вещества, такие как соль, жидкости, такие как спирт, и газы, такие как азот и кислород, могут растворяться в воде.

Страница № 56:
Вопрос 8:

Осаждение и декантация используются для разделения несмешивающихся жидкостей
(a).
(b) твердое вещество, растворимое в жидкости.
(c) твердое вещество, не растворимое в жидкости.
(d) нерастворимое твердое вещество из жидкости, где твердое вещество тяжелее жидкости.

Ответ:

(d) нерастворимое твердое вещество из жидкости, где твердое вещество тяжелее жидкости

Декантация и седиментация могут использоваться для отделения твердых частиц от жидкости, когда твердое вещество тяжелее жидкости и нерастворимо в жидкости.

Страница № 56:
Вопрос 9:

Обычно используемым химическим веществом для загрузки является водород
(a).
(б) квасцы.
(в) сера.
(г) соль поваренная.

Ответ:

(b) квасцы

Квасцы — это обычно используемый химикат для загрузки с целью ускорения седиментации.

Страница № 56:
Вопрос 10:

Какой метод вы будете использовать, чтобы получить только чистое твердое вещество из раствора в воде?
(a) выпаривание
(b) дистилляция
(c) фильтрация
(d) ни один из этих

Ответ:

(a) испарение

Испарение — это процесс, используемый для получения чистого твердого вещества из его раствора в воде.

Страница № 56:
Вопрос 1:

Вокруг нас очень мало чистых элементов или соединений. Большинство веществ представляют собой смеси. Правда или ложь?

Ответ:

Действительно, большинство веществ вокруг нас существует в смешанном состоянии, а не в виде чистых элементов или соединений.

Страница № 56:
Вопрос 2:

Составляющие всех смесей равномерно распределены по смесям.Правда или ложь?

Ответ:

Неверно, компоненты гетерогенной смеси неравномерно распределены по смеси.

Страница № 56:
Вопрос 3:

Является ли раствор сахара в воде однородным или неоднородным?

Ответ:

В случае раствора сахара сахар равномерно распределяется в воде.Таким образом, сахарный раствор представляет собой однородный раствор.

Страница № 56:
Вопрос 4:

Назовите одно свойство соединения, которое остается фиксированным и может быть использовано для проверки наличия в составе соединений примесей.

Ответ:

Точка плавления и точка кипения — это два свойства соединения, которые можно использовать для проверки его чистоты, поскольку они остаются фиксированными.

Страница № 56:
Вопрос 5:

Мы должны разделить смесь на составляющие, потому что нам полезны только чистые соединения. От смесей особого толка нет. Правда или ложь?

Ответ:

Ложь, смеси нам тоже очень пригодятся. Например, воздух, которым мы дышим, также представляет собой смесь газов.

Страница № 56:
Вопрос 6:

Можно ли разделить смесь опилок и воды путем осаждения и декантации?

Ответ:

Осаждение и декантацию нельзя использовать для отделения опилок от воды, потому что опилки не тяжелее воды.Осаждение и декантацию можно использовать для отделения нерастворимого твердого вещества от жидкости, только если твердое вещество тяжелее жидкости.

Страница № 56:
Вопрос 7:

Какой метод дает лучшее отделение нерастворимого твердого вещества от жидкости — осаждение и декантация или фильтрация?

Ответ:

Нерастворимое твердое вещество можно более эффективно отделить от жидкости путем фильтрации.Фильтрация может отделять твердые частицы, которые также легче жидкости, и происходит быстрее, поскольку жидкость не нужно оставлять стоять.

Страница № 56:
Вопрос 8:

Назовите метод, который используется для ускорения седиментации.

Ответ:

Метод, используемый для ускорения осаждения, известен как загрузка.

Страница № 56:
Вопрос 9:

Какой метод отделения используется для отделения зерен пшеницы от половы?

Ответ:

Зерна пшеницы отделяются от плевел методом веяния.

Страница № 56:
Вопрос 10:

Назовите метод, используемый для получения соли из морской воды.

Ответ:

Соль получают из морской воды испарением морской воды.

Страница № 56:
Вопрос 11:

Какой метод вы будете использовать, чтобы получить чистую жидкость из раствора, содержащего несколько растворимых твердых веществ в жидкости?

Ответ:

Дистилляция может использоваться для получения чистой жидкости из раствора, содержащего несколько растворимых твердых веществ в жидкости.

Страница № 56:
Вопрос 12:

Какой аппарат используется для охлаждения паров жидкости во время перегонки?

Ответ:

Конденсатор Либиха используется для охлаждения паров жидкости во время перегонки.

Страница № 56:
Вопрос 13:

Какой аппарат используется для разделения двух несмешивающихся жидкостей?

Ответ:

Две несмешивающиеся жидкости можно разделить с помощью делительной воронки.

Страница № 56:
Вопрос 14:

Растворимость большинства твердых веществ в воде увеличивается с повышением температуры. Правда или ложь?

Ответ:

Верно. Растворимость большинства твердых веществ в воде увеличивается с повышением температуры.

Страница № 57:
Вопрос 1:

Вам дают образец воды.Как вы узнаете, чистая она или нет?

Ответ:

Чтобы узнать, чистая вода или нет, мы можем измерить температуру кипения образца. Если образец закипает при температуре 100- o C, значит вода чистая.

Страница № 57:
Вопрос 2:

Каков принцип разделения смесей?

Ответ:

Принцип разделения смеси основан на особом свойстве одного из компонентов смеси.Например, куски железа, смешанные с песком, можно отделить с помощью магнита, поскольку железо притягивается к магниту.

Страница № 57:
Вопрос 3:

Чтобы использовать сито для отделения песка и семян риса, каким должен быть размер отверстий сита по сравнению с размером частиц песка и семян риса?

Ответ:

Для отделения песка от семян риса отверстия в сите должны быть больше, чем частицы песка, и меньше, чем семена риса, чтобы частицы песка могли легко проходить через сита, оставляя после себя семена риса.

Страница № 57:
Вопрос 4:

Какое свойство фильтровальной бумаги используется для отделения нерастворимого твердого вещества от жидкости?

Ответ:

Нерастворимое твердое вещество можно отделить от жидкости с помощью фильтровальной бумаги, поскольку фильтровальная бумага позволяет жидкости проходить через нее, в то время как твердое вещество не проходит через фильтровальную бумагу.

Страница № 57:
Вопрос 5:

Мы всегда говорим о насыщенном растворе при определенной температуре? Почему указана температура?

Ответ:

Растворимость твердого вещества в растворе увеличивается с повышением температуры.Если повысить температуру насыщенного раствора, раствор станет ненасыщенным. Таким образом, температура играет важную роль в определении того, является ли раствор насыщенным или нет.

Страница № 57:
Вопрос 6:

При каких условиях можно использовать ручной сбор для разделения компонентов смеси?

Ответ:

Ручной сбор можно использовать для разделения компонентов смеси только в том случае, если компоненты смеси хорошо видны и могут быть разделены.Например, камни можно отделить от риса вручную.

Страница № 57:
Вопрос 7:

Почему предметы на расстоянии видны более отчетливо после дождя?

Ответ:

Во время дождя частицы пыли в воздухе намокают и оседают на земле. В результате после дождя воздух становится чище, а объекты на расстоянии видны более отчетливо.

Страница № 57:
Вопрос 8:

Процесс добавления квасцов в воду для ускорения осаждения называется «загрузкой». Почему процесс получил такое название?

Ответ:

Когда квасцы добавляются в воду, они растворяются в воде и утяжеляют взвешенные примеси, заставляя их быстро оседать. Поэтому добавление квасцов называется загрузкой.

Страница № 57:
Вопрос 1:

В чем разница между гомогенными и гетерогенными смесями? Приведите пример каждой смеси.

Ответ:
Гомогенная смесь Гетерогенная смесь
Компоненты смеси равномерно распределены в смеси. Компоненты смеси неравномерно распределены в смеси.
Пример: растворенная в воде соль представляет собой гомогенную смесь. Пример: Смешанные с песком железные опилки представляют собой неоднородную смесь.
Страница № 57:
Вопрос 2:

Перечислите три свойства смесей.

Ответ:

Ниже приведены свойства смесей:

  1. Составляющие смеси могут присутствовать в любом соотношении.
  2. Составные части смеси сохраняют свои индивидуальные свойства. Молекулы компонентов не изменяются. Например, когда железо и сера смешиваются вместе, их свойства не меняются.
  3. Составные части смеси легко разделяются. Например, камни в рисе можно легко отделить визуальным наблюдением.
Страница № 57:
Вопрос 3:

Приведите две ситуации, когда необходимо разделить смесь на составляющие.

Ответ:

Разделение смеси на составляющие необходимо в следующих случаях:

  1. Удаление нежелательных составляющих: например, желательно удалить мелкие камни в рисе или отделить чайные листья от чая.
  2. Для получения полезных веществ: Сливочное масло получают сбиванием творога. Таким же образом сырая нефть может быть разделена на различные масла, такие как бензин, керосин, дизельное топливо и т. Д.
Страница № 57:
Вопрос 4:

Нарисуйте помеченную диаграмму, чтобы показать устройство, используемое для фильтрации смеси жидкости и нерастворимого твердого вещества.

Ответ:

Устройство, используемое для фильтрации смеси жидкости от нерастворимого твердого вещества, показано ниже:

Страница № 57:
Вопрос 5:

Нарисуйте помеченную схему аппарата, используемого для отгонки чистой воды из раствора.

Ответ:

Схема аппарата для отгонки чистой воды из раствора

Страница № 57:
Вопрос 6:

Как отделить смесь травы, гальки и песка?

Ответ:

Чтобы отделить смесь травы, гальки и песка, сначала нужно пропустить смесь через фильтр тонкой очистки.Песок будет проходить через фильтр, оставляя траву и гальку на фильтре.
Затем мы можем веять траву и гальку, поскольку они имеют разный вес, чтобы отделять их друг от друга.

Страница № 57:
Вопрос 7:

Объясните с помощью схемы, как можно разделить смесь масла и воды.

Ответ:

Масло и воду можно разделить с помощью делительной воронки.Схема эксперимента показана на рисунке:

Смесь масла и воды наливают в делительную воронку и оставляют на некоторое время. Масло скапливается в верхней части воронки, а вода — у ее дна, как показано на рисунке. Если запорный кран внизу открыт, вода вытекает из воронки и может собираться в стакан. Если запорный кран закрыт, когда масло достигает уровня запорного крана, в воронке будет масло, а в стакане — вода.

Страница № 57:
Вопрос 8:

Почему вода считается важным растворителем?

Ответ:

Вода считается важным растворителем, потому что:

  1. Вода может растворять большое количество веществ, важных для жизни.
  2. Отходы также растворяются в воде, прежде чем они могут быть выведены из организма.
  3. Большинство химических реакций, происходящих в организме, происходят в присутствии воды.
  4. Такие газы, как кислород и диоксид углерода, растворимы в воде. Это делает возможной водную жизнь.
Страница № 57:
Вопрос 15:

Может ли вода растворять жидкости и газы?

Ответ:

Да, вода растворяет такие жидкости, как спирт или молоко.Он также может растворять такие газы, как кислород, азот и т. Д.

Страница № 57:
Вопрос 16:

Растворимость поваренной соли быстро увеличивается с повышением температуры. Правда или ложь?

Ответ:

Верно. Растворимость поваренной соли быстро увеличивается с повышением температуры.

Страница № 57:
Вопрос 1:

Бутылка содержит соль или мел.Как вы узнаете, что в нем есть, не пробуя его на вкус?

Ответ:

Соль растворима в воде, а мел — нет. Итак, если вы добавите соль в воду, она растворится и образует прозрачную жидкость. Однако мел не растворяется в воде, и меловая пыль остается в воде.

Страница № 57:
Вопрос 2:

Подумайте и напишите, как вы разделите смесь воды, песка, соли и железа.Возможно, вам придется использовать метод, не описанный в этой главе, для отделения песка и железной опилки. Подумайте — какое свойство железа для этого можно использовать?

Ответ:

Для разделения компонентов смеси мы можем использовать следующие шаги:

  1. Первый шаг — пропустить смесь через фильтровальную бумагу. Это позволит отделить смесь песка и железных опилок от солевого раствора.
  2. Теперь проведите магнитом над смесью песка и железных опилок.Железные опилки будут притягиваться к магниту.
  3. Наконец, выпарить солевой раствор и собрать пары. Сконденсируйте пар, чтобы получить воду, а соль останется.
Страница № 57:
Вопрос 3:

Смесь соли и сахара получают путем их измельчения. Изменяются ли молекулы соли и сахара?

Ответ:

Нет, молекулы соли и сахара остаются такими, как есть, даже когда соль и сахар измельчают для получения смеси.

Страница № 57:
Вопрос 4:

Готовят насыщенный раствор сахара в воде. Температура воды понижена на 5 ° C. Будет ли раствор оставаться насыщенным? Назови причины.

Ответ:

Если раствор насыщен, это означает, что больше твердого вещества не растворяется в растворе при этой температуре. Если снизить температуру, растворимость сахара еще больше снизится, и раствор останется насыщенным.

Страница № 57:
Вопрос 5:

Морская вода содержит растворенную соль. Вы знаете, что чистую воду можно получить из морской воды дистилляцией. Однако этот метод не используется для получения питьевой воды из морской воды. Как вы думаете, в чем причина?

Ответ:

Морская вода содержит растворенную соль, поэтому чистую воду можно получить дистилляцией.Однако энергия, необходимая для нагрева больших количеств морской воды, чтобы сделать этот процесс широко применимым, слишком дорога. Кроме того, требуемые конденсаторы очень дороги, если они должны быть достаточно большими, чтобы перегонять большое количество морской воды. В результате для получения питьевой воды из морской воды дистилляция не используется.

Просмотреть решения NCERT для всех глав класса 6

.

Оставить комментарий